Download as pdf or txt
Download as pdf or txt
You are on page 1of 56

ALLEN Basic Mathematics

BASIC MATHEMATICS IN PHYSICS


TRIGONOMETRY
Angle :
Consider a revolving line OP. Suppose that it revolves in anticlockwise direction
starting from its initial position OX .
The angle is defined as the amount of revolution that the revolving line makes
with its initial position.
From figure the angle covered by the revolving line OP is θ = ∠POX
The angle is positive :
If it is traced by the revolving line in anticlockwise direction
and is negative :
If it is covered in clockwise direction.
1° = 60' (minute)
1' = 60" (second)
π
1 right angle = 90° (degrees) also 1 right angle = rad (radian)
2
One radian is the angle subtended at the centre of a circle by an arc of the
circle, whose length is equal to the radius of the circle.
r
o
180
1 rad = ≈ 57° 17' 45" ≈ 57.3° θ = 1 rad l =r
π
Trigonometrical ratios (or T ratios)
Let two fixed lines XOX' and YOY' intersecting at right angles to each other at point O.
Then,
(i) Point O is called origin.
(ii) XOX' known as X-axis and YOY' are Y-axis.
(iii) Portions XOY, YOX', X'OY' and Y'OX are called I, II, III and IV quadrant respectively.
Consider that the revolving line OP has traced out angle θ
(in I quadrant) in anticlockwise direction.From P, perpendicular PM on OX.
Then, side OP (in front of right angle) is called hypotenuse,side MP (in front
of angle θ) is called opposite side or perpendicular and side OM (making
angle θ with hypotenuse) is called adjacent side or base.
The three sides of a right angled triangle are connected to each other through
six different ratios, called trigonometric ratios or simply T-ratios :

perpendicular MP base OM
sin θ = = cos θ = =
hypotenuse OP hypotenuse OP

perpendicular MP base OM
tan θ = = cot θ = =
base OM perpendicular MP

hypotenuse OP hypotenuse OP
sec θ = = cosec θ = =
base OM perpendicular MP
It can be easily proved that :
1 1 1
cosec θ = , sec θ = , cot θ = ,
sin θ cos θ tan θ
sin2θ + cos2θ = 1, 1 + tan2θ = sec2θ, 1 + cot2θ = cosec2θ

E 1
JEE ( Main) - Physics ALLEN

SOLVED EXAMPLE

3 P
Ex. Given sin θ = . Find all the other T-ratios, if θ lies in the first quadrant.
5
3 5
Sol. In ∆ OMP , sin θ = so MP = 3 and OP = 5 3
5
Q OM = (5)2 − (3)2 = 25 − 9 = 16 = 4 θ
O M
OM 4 MP 3
Now, cos θ = = tan θ = =
OP 5 OM 4
OM 4 OP 5 OP 5
cot θ = = sec θ = = cosec θ = =
MP 3 OM 4 MP 3
Question For Practice

5
Q. If sec θ = , find all the other T-ratios.
3
4 3 4 3 5
Ans. sinθ = , cosθ = , tanθ = , cotθ = , cosecθ =
5 5 3 4 4
The T-ratios of a few standard angles ranging from 00 to 1800
Angle (θ) 0° 30° 45° 60° 90° 120° 135° 150° 180°
1 1 3 3 1 1
sin θ 0 1 0
2 2 2 2 2 2

3 1 1 1 1 3
cos θ 1 0 – − − –1
2 2 2 2 2 2
1 1
tan θ 0 1 3 ∞ – 3 –1 − 0
3 3

Angle 37° 53°


sin 3/5 4/5
5 cm 37 °
cos 4/5 3/5 4 cm
tan 3/4 4/3 53°
3 cm

sin (90° + θ) = cos θ sin (180° – θ) = sin θ sin (–θ) = – sin θ sin (90° – θ) = cos θ
cos (90° + θ) = – sin θ cos (180° – θ) =– cos θ cos (–θ) = cos θ cοs (90° – θ) = sin θ
tan (90°+ θ) =– cot θ tan (180° – θ) =– tan θ tan (–θ) = – tan θ tan (90° –θ) = cotθ

sin (180° + θ) = – sin θ sin (270° – θ) = – cos θ sin (270° + θ) = – cos θ sin (360° – θ) = – sin θ
cos (180° + θ) = – cos θ cos(270° – θ)= – sinθ cos (270° + θ) = sin θ cos (360° – θ) = cos θ
tan (180° +θ) = tan θ tan (270° – θ) = cot θ tan (270° + θ) = – cot θ tan (360° – θ) = – tan θ

θ → 0 ⇒ sin θ ≈ θ, tanθ ≈ θ, cos θ ≈ 1, tan θ ≈ sinθ If θ in radian

2 E
ALLEN Basic Mathematics

SOLVED EXAMPLE
Ex. Find the value of (i) cos (–60°) (ii) tan 210° (iii) sin 300° (iv) cos 120°
1
Sol. (i) cos (–60°) = cos 60° =
2

1
(ii) tan 210° = tan (180° + 30°) = tan 30° =
3

3
(iii) sin 300° = sin (270° + 30°) = – cos 30° = −
2

1
(iv) cos 120° = cos (180° – 60°) = – cos60° = –
2

Question For Practice


Q. Find the values of :
(i) tan (–30°) (ii) sin 120° (iii) sin 135° (iv) cos 150° (v) sin 270° (vi) cos 270°

1 3 1 3
Ans. (i) − (ii) (iii) (iv) − (v) –1 (vi) 0
3 2 2 2

A few important trigonometric formulae


sin (A + B) = sin A cos B + cos A sin B cos (A + B) = cos A cos B – sin A sin B
sin ( A – B) = sin A cos B – cos A sin B cos (A – B) = cos A cos B + sin A sin B

tan A + tan B tan A − tan B


tan (A + B ) = tan (A – B) =
1 − tan A tan B 1 + tan A tan B
sin 2 A = 2 sin A cos A cos 2 A = cos2 A – sin2 A

2 tan A
tan 2 A = cos 2 A = 2 cos2 A – 1 = 1 – 2sin2 A
1 − tan2 A

DIFFERENTIATION
Function :
Constant: A quantity, whose value remains unchanged during mathematical operations, is called a constant
quantity. The integers, fractions like π ,e, etc are all constants.
Variable: A quantity, which can take different values, is called a variable quantity. A variable is usually
represented as x, y, z, etc.
Function: A quantity y is called a function of a variable x, if corresponding to any given value of x, there exists
a single definite value of y. Then 'y is function of x' is represented as y = f (x)
For example, consider that y is a function of the variable x which is given by
y = 3x2 + 7x + 2
If x = 1, then y = 3 (1)2 + 7(1) + 2 = 12
and when x = 2, y = 3 (2)2 + 7(2) + 2 = 28
Therefore, when the value of variable x is changed, the value of the function y also changes but corresponding
to each value of x, we get a single definite value of y. Hence, y = 3x2 + 7x + 2 represents a function of x.

E 3
JEE ( Main) - Physics ALLEN
dy
Physical meaning of :
dx
1. The ratio of small change in the function y and the variable x is called the average rate of change of y w.r.t.x.
∆s
For example, if a body covers a small distance ∆s in small time ∆ t, then average velocity of the body, vav = .
∆t
Also, if the velocity of a body changes by a small amount ∆v in small time ∆t, then average acceleration of the
∆v
body, aav =
∆t
∆y ∆y dy
2. When ∆ x → 0 The limiting value of is Lim =
∆x ∆x →0 ∆x dx
It is called the instantaneous rate of change of y w.r.t. x. y
The differentiation of a function w.r.t. a variable implies the instantaneous
rate of change of the function w.r.t. that variable.
∆s ds
Like wise, instantaneous velocity of the body, (v) = Lim = and
∆t →0 ∆t dt θ ∆y
θ ∆x
∆v dv x
instantaneous acceleration of the body (a) = Lim =
∆t →0 ∆t dt

Theorems of differentiation :
d
1. If c = constant, (c) = 0
dx
dy d du
2. y = c u, where c is a constant and u is a function of x, = (cu) = c
dx dx dx
dy d du dv dw
3. y = u ± v ± w, where, u, v and w are functions of x, = ( u ± v ± w) = ± ± dx
dx dx dx dx
dy d dv du
4. y = u v where u and v are functions of x, = (uv) = u +v
dx dx dx dx
du dv
u v −u
dy d u
5. y=
v
, where u and v are functions of x, = = dx 2 dx
dx dx v v
dy d n
6. y = xn, n real number, = (x ) = nxn–1
dx dx

SOLVED EXAMPLE

dy
Ex. Find , when (i) y= x (ii) y = x5 + x4 + 7 (iii) y = x2 + 4x –1/2 – 3x–2
dx

dy d d 1/ 2 1 1 −1 / 2 1
Sol. (i) y = x ⇒ = ( x) = (x ) = x1 / 2−1 = x =
dx dx dx 2 2 2 x

dy d 5 d 5 d 4 d
(ii) y = x5 + x4 + 7 ⇒ = (x + x4 +7) = (x ) + (x ) + (7) = 5x4 + 4x3 + 0 = 5x4 + 4x3
dx dx dx dx dx
(iii) y = x2 + 4x–1/2 – 3x–2
dy d 2 d 2 d d
= (x + 4x–1/2 – 3x–2) = (x ) + (4x–1/2) – (3x–2)
dx dx dx dx dx

d 2 d –1/2 d –2  1
= (x ) + 4 (x ) –3 (x ) = 2x + 4  −  x–3/2 –3(–2)x–3 = 2x – 2x–3/2 + 6x–3
dx dx dx 2

4 E
ALLEN Basic Mathematics
Ex. Find the derivatives of the following :
9x 5
(i) (x3 –3x2 + 4) (4x5 + x2 – 1) (ii)
x−3
Sol. (i) Let y = (x3 – 3x2 + 4) (4x5 + x2 – 1)
dy d d 3
= (x3 – 3x2 + 4) (4x5 + x2 – 1) + (4x5 + x2 – 1) (x – 3x2 + 4)
dx dx dx

d d 2 d  d d d 
= (x3 – 3x2 + 4)  (4x ) + (x ) − (1) + (4x5 + x2 – 1)  (x 3 ) − (3x 2 ) +
5
(4)
 dx dx dx   dx dx dx 

 d 5 d 2 d  d d d 
= (x3 – 3x2 + 4) 4 (x ) + (x ) − (1)  + (4x5 + x2 – 1)  (x 3 ) − 3 (x2 ) + (4)
 dx dx dx   dx dx dx 
= (x3 – 3x2 + 4)[4 × 5x4 + 2x – 0] + (4x5 + x2 – 1) [3x2 – 3 × 2x + 0]
= (x3 – 3x2 + 4) (20x4 + 2x) + (4x5 + x2 –1) (3x2 –6x)
= 2x(10x3 + 1) (x3 – 3x2 + 4) + 3x (x –2) (4x5 + x2 – 1)

9x 5
(ii) Let y =
x−3

d d
dy ( x − 3) dx (9x5 ) − 9x5 dx (x − 3)
=
dx (x − 3)2

d
( 3)  (x − 3) × 9 × 5x 4 − 9x5 (1 − 0)
d 5 d
(x − 3) × 9 (x ) − 9x 5  (x) −
= dx  dx dx =
(x − 3)2 (x − 3)2

45x 5 − 135x 4 − 9x 5 36x 5 − 135x 4 9x 4 (4x − 15)


= = =
(x − 3)2 (x − 3)2 (x − 3)2

Question For Practice

dy
Q1. Find for the following :
dx

(i) y = x7/2 (ii) y = x–3 (iii) y=x


(iv) y = x5 + x3 + 4x1/2 + 7 (v) y = 5x4 + 6x3/2 + 9x (vi) y = ax2 + bx + c
1
(vii) y = 3x5 – 3x – (viii) y=(x – 1) (2x + 5) (ix) y=(9x3 – 8x + 7) (3x5 + 5)
x

1 3x + 4 x2
(x) y= (xi) y= (xii) y=
2x + 1 4x + 5 x +1
3

ds
Q2. Given s = t2 + 5t + 3, find .
dt

1 2 ds
Q3. If s = ut + at , where u and a are constants. Obtain the value of .
2 dt
Q4. The area of a blot of ink is growing such that after t seconds, its area is given by A = (3t2+7) cm2 . Calculate the
rate of increase of area at t= 5 second.
Q5. The area of a circle is given by A = π r2 , where r is the radius. Calculate the rate of increase of area
w.r.t. radius.

E 5
JEE ( Main) - Physics ALLEN
Ans.
7 5/2
1. (i) x (ii) – 3x– 4 (iii) 1
2
(iv) 5x4 + 3x2 + 2x–1/2 (v) 20x3 + 9x1/2 + 9 (vi) 2ax + b
1
(vii) 15x4 – 3 + (viii) 4x + 3 (ix) 216 x7 – 144 x5 + 105 x4 + 135 x2 – 40
x2

2 1 2x − x 4
(x) – (xi) – (xii)
(2x + 1)2 (4x + 5)2 (x 3 + 1)2
2. 2t + 5 3. u + at 4. 30 cm2 s –1
5. 2 πr

Some Important Formulas :


d d d
1. (sin x) = cos x 2. (cos x) = – sin x 3. (tan x) = sec2 x
dx dx dx
d d d
4. (cot x) = – cosec2 x 5. (sec x) = sec x tan x 6. (cosec x) = – cosec x cot x
dx dx dx
d 1 d
7. (logex) = 8. (ex) = ex
dx x dx
Maxima and Minima : y

dy d2 y
For minimum value of y, = 0 and = + ve
dx dx2 Maxima
y=f(x)
2
dy d y
For maximum value of y, = 0 and = − ve
dx dx2 Minima
x
SOLVED EXAMPLE

Ex. Find minimum value of y = 25x2 + 5 - 10x


dy 1
Sol. For maximum/minimum value = 0 ⇒ 50x - 10 = 0 ⇒ x =
dx 5

1 d2 y
Now at x = , = 50 which is positive So
5 dx2

2
1 1
y min = 25   + 5 − 10   = 1 + 5 − 2 = 4
5 5
INTEGRATION
In integral calculus, the differential coefficient of a function is given. We are required to find the function.
Integration is basically used for summation . ∑ is used for summation of discrete values, while ∫ sign is used for
continous function.
dI
If I is integration of f(x) with respect to x then I = ∫ f(x) dx [we can check = f(x) ]
dx
d n+1
Let us proceed to obtain intergral of xn w.r.t. x. (x ) = (n +1)xn
dx
Since the process of integration is the reverse process of differentiation,
x n +1
∫ (n + 1)x n dx = x n+1 or (n + 1) ∫ x n dx = x n+1 or ∫ x n dx =
n +1
The above formula holds for all values of n, except n = –1.

6 E
ALLEN Basic Mathematics

1 d 1 1
It is because, for n = –1,
x ∫
dx Q x n dx =
dx
(loge x) =
x
∴ ∫ dx = loge x
∫ x
x −1dx = ∫
Similarly, the formula for integration of some other functions can be obtained if we know the differential
coefficients of various functions.

Few basic formulae of integration :


Following are a few basic formulae of integration :
x n +1 d
1. ∫ x n dx =
n +1
+ c , Provided n ≠ – 1 2. ∫ sin xdx = − cos x + c (Q
dx
(cos x)= – sin x)

d 1 d 1
3. ∫ cos xdx = sin x + c (Q
dx
sin x = cos x) 4. ∫ x
dx = loge x + c (Q
dx
(loge x)= )
x
d x
5. ∫ e x dx = e x + c (Q
dx
(e )= ex)

SOLVED EXAMPLE
11/2
Ex. Integrate w.r.t. x. : (i) x (ii) x–7 (iii) xp/q
x11 / 2+1 2 13/2 x −7+1 1 −
∫ ∫x +c = – x 6 +c
−7
Sol. (i) x11 / 2 dx = +c= x +c (ii) dx =
11
+1 13 −7 + 1 6
2
p
+1
p
xq q
(iii) ∫ x dx = p +c= x(p + q) / q + c
q
p+q
+1
q

1
Ex. Evaluate ∫ (x2 − cos x + ) dx
x
1 x2+1 x3
Sol. = ∫ x2dx − ∫ cos xdx + ∫
x
dx =
2+1
– sin x + loge
x + c =
3
– sin x + loge x + c
Question For Practice
Q. Evaluate the following integrals :
∫ ∫ x − 2 dx ∫ (3x −7 + x −1 ) dx
3
(i). x15 dx (ii). (iii).
2
 1   1  a b
(iv). ∫  x+

 dx
x
(v). ∫  x +  dx
 x
(vi). ∫  2 +  dx (a and b are constant)
x x
x16 –1/2
x −6
Ans. (i). +c (ii). – 2x +c (iii). – + loge x + c
16 2
x2 x2 a
(iv). + 2x + loge x + c (v). + loge x + c (vi). – + b loge x + c
2 2 x

Definite Integrals : y
When a function is integrated between a lower limit and an upper limit,
d
it is called a definite integral. If
dx
(f(x)) = f ' (x), then ∫ f ' (x) dx is called

b x2
indefinite integral and ∫ a
f ' (x) dx is called difinite integral Here, a and b are ∫x 1
ydx
x
x1 x2
called lower and upper limits of the variable x.
After carrying out integration, the result is evaluated between upper and lower limits as explained below :
b

b
f ' (x) dx = f ( x ) = f(b) – f(a)
a a

x2
Area bounded by the curve = ∫x1
ydx

E 7
JEE ( Main) - Physics ALLEN
SOLVED EXAMPLE
5
Ex. Evaluate the integral : ∫ 1
x2 dx

5
5  x3  1 5 1 1 124
∫ x dx =   =  x3  1 = ((5)2 − (1)3 ) =
2
Sol. (125 – 1) =
1  3 1 3 3 3 3
Question For Practice
Q. Evaluate the following integrals
∞ q1q2 ∞
GMm r2 v −1

∫R ∫r ∫u ∫0
2
(i) dx (ii) −k dx (iii) Mv dv (iv) x dx
x2 1 r 2

π π π
2

∫−
2 2

(v) ∫0 sin x dx (vi) ∫0 cos x dx (vii) π


2
cos x dx

GMm  1 1 1
Ans. (i) (ii) kq1q2  −  (iii) M (v2 – u2) (iv) ∞
R  r2 r1  2
(v) 1 (vi) 1 (vii) 2

AVERAGE OF A VARYING QUANTITY


x2 x2

< y >= y =
∫ x1
ydx
=

x1
f(x)dx
If y = f(x) then average value of this function from x = x1 to x = x2 x2 x2
∫ x1
dx ∫x1
dx

1
Ex. Kinetic energy of a particle executing S.H.M. is K = mω2 (a2 − x2 ) calculate average value of kinetic
2
energy from x = 0 to x = a
a 1
∫ Kdx = ∫
a
mω2 (a2 − x2 )dx
0 2 1
Sol. K= 0
a
= mω2 a2
∫ dx 0
a 3

ALGEBRA

Quadratic equation and its solution :


An algebraic equation of second order (highest power of the variable is equal to 2) is called a quadratic equation.
Equation ax2 + bx + c = 0 is the general quadratic equation.
The general solution of the above quadratic equation or value of variable x is

− b ± b2 − 4ac − b + b2 − 4ac − b − b2 − 4ac


x= ⇒ x1 = and x2 =
2a 2a 2a
Ex. 2x2 + 5x – 12 = 0
Sol. By comparision with the standard quadratic equation a = 2, b = 5 and c = –12

−5 ± (5)2 − 4 × 2 × ( −12) −5 ± 121 −5 ± 11 +6 −16 3


x= = = = , or x= ,–4
2×2 4 4 4 4 2
Question For Practice
Q. Solve for x : (i) 10x2 – 27x + 5 = 0 (ii) pqx2 – (p2 + q2) x + pq = 0
5 1 p q
Ans. (i) , (ii) ,
2 5 q p

8 E
ALLEN Basic Mathematics
Binomial theorem :
n ( n − 1) n ( n − 1) ( n − 2)
It states that (1 + x )n = 1 + nx + 2 !
x² +
3 !
x³ + .....

where x is less than 1 and n is any number, positive, negative or a fraction.


Here, 2 ! = 2 × 1, 3! = 3 × 2 × 1 and in general, n ! = n (n – 1) (n – 2) .......1
2!, 3! or n! are read as factorial 2, factorial 3 or factorial n respectively.
Note : In case, x is very small, then terms containing higher powers of x can be neglected. In such a case,
n –n
(1 + x) = 1 + nx and (1 + x) = 1 – nx
n –n
(1 – x) = 1 – nx and (1 – x) = 1 + nx
x x2 x 3 –x x2 x3
Exponential Expansion e =1+x+ + + ........ e = 1 – x + − + ........
2! 3! 2! 3!
Componendo and dividendo theorem :
p a p−q a−b
If = then by componendo and dividendo theorem =
q b p+q a+b
Determinant :
a b a b
D= = ad − bc ,
c d

−3 3 2 −4
e.g.# (a) = 12 , = −6
−5 1 −3 3

a1 a2 a3 c d
b1 b2 b3 b2 b 3 b b b b
D= = a1 − a2 1 3 + a 3 1 2
c1 c2 c3 c2 c 3 c1 c3 c1 c2

+ − +
5 4 3 1 6 2 6 2 1
e.g. (b) 2 1 6 =5 −4 +3 = 5 (9 – 48) – 4(18 – 42) + 3(16 – 7)
8 9 7 9 7 8
7 8 9

Exponentials and roots :


1 –n 1/n n/m n 1/m
x° = 1 =x x = n
x x = (x ) = m
xn
xn
n
 x xn
n
(xy) = x y
n n
 y  = = xn y −n nm
(x ) = x
n ×m
x
1/n
y
1/n
= (xy)
1/n
yn
1/ n
 x x1 / n
= (x ) = ( x)
1/ m 1 / mn 1 1
1/ n
 y  y1 / n
3
x = x3 x = x2

SOLVED EXAMPLE

5/2
2/3 3/2  1
Ex. Find (i) 8 , (ii) (256) and   .
 2

( )
2/3 2 1/3 1/3 3 1/3 3/2 3 3
Sol. (i) 8 = (8 ) = (64) = (4 ) =4 (ii) (256) = 256 = (16) = 4096

1 1
 1
5/2
  1 5  2  1  2 1 1 0.25
(iii)   =    =   = = = = 0.18
 2 
 2   32  2 × 16 4 2 1.41

E 9
JEE ( Main) - Physics ALLEN
Logarithm :
(a) Product formula log mn = log m + log n loge x = lnx (base e)
m
(b) Quotient formula log = log m – log n logx = log10 x (base 10)
n
n
(c) Power formula log m = n log m e = 2.718
In physics the formula involving a logarithmic function, the base to the logarithm is always 'e', where e is a
constant having value = 2.718.
GEOMETRY
Formulae for determination of area :
1. Area of a square = (side)2
2. Area of rectangle = length × breadth

1
3. Area of a triangle = base × height
2

1
4. Area of a trapezoid = × (distance between parallel sides) × (sum of parallel sides)
2
5. Area enclosed by a circle = π r2 (r = radius)
6. Surface area of a sphere = 4π r2 (r = radius)
7. Area of a parallelogram = base × height
8. Area of curved surface of cylinder = 2π rl (r = radius and l = length)
9. Area of ellipse = π ab (a and b are semi major and semi minor axes respectively)
10. Surface area of a cube = 6(side)2
11. Total surface area of a cone = πr2+πrl where πrl = πr r 2 + h2 = lateral area

Formulae for determination of volume :


t
1. Volume of a rectangular slab = length × breadth × height = a b t
2. Volume of a cube = (side)3 a
4 3 b
3. Volume of a sphere = πr (r = radius)
3
4. Volume of a cylinder = π r2l (r = radius and l is length)

1 2
5. Volume of a cone = πr h (r = radius and h is height)
3

22 1
Note : π = = 3.14 ; π2 = 9.8776 ≈ 10 and = 0.3182.
7 π

SOLVED EXAMPLE

Ex. Calculate the shaded area. 4


Sol. Shaded area = Area of ellipse = πab
3
Here a = 6 – 4 = 2 and b = 4 – 3 = 1
⇒ Area = π × 2 × 1 = 2π units 2

4 6 8

Ex. Calculate the volume of given disk. R=1m


Sol. Volume = πR2t = (3.14) (1)2 (10–3)2 = 3.14 × 10–6 m3 t=1mm

10 E
ALLEN Basic Mathematics

SOME STANDARD GRAPHS AND THEIR EQUATIONS

y y A y

-c
Straight line c

x
x+

m
θ
=
O x =m
y y = mx y
c Straight line
Straight line
c
θ x θ x
O O
y
y y
2 2
y = - mx + c x = ky x = -ky
Straight line x
x
O O
Parabola
θ Parabola
x
O

y y Hyperbola
y O 1
x x y
Parabola
2 y = -kx2
y = kx xy = constant

Parabola
x x
O O

y Ellipse y x 2 y2 y
2 2 2 + =1 Exponential Decay
x+y=a a2 b 2 1 -kx
b y=e

O a x
x O
Circle
a a = semi major axis
O x
b = semi minor axis

E 11
JEE ( Main) - Physics ALLEN
UNITS AND DIMENSIONS
PHYSICAL QUANTITIES :
All the quantities those can be measured and which are used to describe the laws of physics are known as
physical quantities.
Classification : Physical quantities can be classified on the following bases :
I. Based on their directional properties
1. Scalars : The physical quantities which have only magnitude but no direction are
called scalar quantities.
e.g. mass, density, volume, time, etc.
2. Vectors : The physical quantities which have both magnitude and direction and
obey laws of vector alzebra are called vector quantities.
e.g. displacement, force, velocity, etc.
II Based on their dependency
1. Fundamental or base quantities : The quantities which do not depend upon other
quantities for their complete definition are known as fundamental or base quantities.
e.g. length, mass, time, etc.
2. Derived quantities : The quantities which can be expressed in terms of the
fundamental quantities are known as derived quantities .
e.g. Speed (=distance/time), volume, accelaration, force, pressure, etc.

GOLDEN KEY POINTS


• Physical quantities can also be classified as dimensional or dimensionless and constant or variable.
• Some physical quantities can not be completely specified even by specifying their magnitude, unit and direction.
These quantities behave neither as a scalar nor as a vector and are called tensors. e.g. Moment of Inertia. It
is not a scalar as it has different values in different directions (i.e.about different axes). It is not a vector as
changing the sense of rotation (i.e. clockwise or anti clockwise) does not change its value.
Question For Practice
Q. Classify the quantities displacement, mass, force, time, speed, velocity, accelaration, moment of intertia, pressure
and work under the following categories :
(a) base and scalar (b) base and vector
(c) derived and scalar (d) derived and vector
Ans. (a) mass, time (b) displacement
(c) speed, pressure, work (d) force, velocity, accelaration
UNITS OF PHYSICAL QUANTITIES
The chosen reference standard of measurement in multiples of which, a physical quantity is expressed is
called the unit of that quantity.
System of Units :
1. FPS or British Engineering system :
In this system length, mass and time are taken as fundamental quantities and their base units are foot
(ft), pound (lb) and second (s) respectively.
2. CGS or Gaussian system :
In this system the fundamental quantities are length, mass and time and their respective units are
centimetre (cm), gram (g) and second (s).
3. MKS system :
In this system also the fundamental quantities are length, mass and time but their fundamental units
are metre (m), kilogram (kg) and second (s) respectively.
4. International system (SI) of units :
This system is modification over the MKS system and so it is also known as Rationalised MKS
system. Besides the three base units of MKS system four fundamental and two supplementary units
are also included in this system.

12 E
ALLEN Basic Mathematics
SI BASE QUANTITIES AND THEIR UNITS
S. No. Physical quantity Unit Symbol
1. Mass kilogram kg
2. Length metre m
3. Time second s
4. Temperature kelvin K
5. Electric current ampere A
6. Luminous intensity candela cd
7. Amount of substance mole mol

While defining a base unit or standard for a physical quantity the following characteristics must be considered :
(i) Well defined (ii) Invariability (constancy)
(iii) Accessibility (easy availability) (iv) Reproducibility
(v) Convenience in use
Classification of Units : The units of physical quantities can be classified as follows :
1. Fundamental or base units :
The units of fundamental quantities are called base units. In SI there are seven base units.
2. Derived units :
The units of derived quantities or the units that can be expressed in terms of the base units are
called derived units.
unit of distance metre
e.g. unit of speed = = = ms −1
unit of time second
Some derived units are named in honour of great scientists.
e.g. unit of force - newton (N), unit of frequency - hertz (Hz), etc.
3. Supplementary units :
In SI two supplementary units are also defined viz. (i) radian (rad) for plane angle and (ii) steradian
(sr) for solid angle.
(i) radian : 1 radian is the angle subtended at the centre of a circle by an arc equal in
length to the radius of the circle.
(ii) steradian : 1 steradian is the solid angle subtended at the centre of a sphere, by
that surface of the sphere which is equal in area to the square of the radius of the sphere.
4. Practical units :
Due to the fixed sizes of SI units, some practical units are also defined for both fundamental and
derived quantities. e.g. light year (ly) is a practical unit of distance (a fundamental quantity) and
horse power (hp) is a practical unit of power (a derived quantity).
Practical units may or may not belong to a particular system of units but can be expressed in any
system of units.
e.g. 1 mile = 1.6 km = 1.6 × 103 m = 1.6 × 105 cm.
5. Improper units :
These are the units which are not of the same nature as that of the physical quantities for which
they are used.
e.g. kg - wt is an improper unit of weight. Here kg is a unit of mass but it is used to measure the
weight (force).
UNITS OF SOME PHYSICAL QUANTITIES IN DIFFERENT SYSTEMS
Type of Physical CGS MKS FPS
Physical Quantity (Originated in France) (Originated in France) (Originated in Britain)
Quantity
Fundamental Length cm m ft
Mass g kg lb
Time s s s
Derived Force dyne newton (N) poundal
Work or erg joule (J) ft - poundal
Energy
Power erg/s watt (W) ft - poundal/s

E 13
JEE ( Main) - Physics ALLEN
Conversion factors :
To convert a physical quantity from one set of units to the other, the required multiplication factor is called
conversion factor.
Magnitude of a physical quantity = numeric value (n) × unit (u)
While conversion from one set of units to the other the magnitude of the quantity must remain same. Therefore
1
n1u1 = n2u2 or nu = constant or n∝
u
That is the numeric value of a physical quantity is inversely proportional to the base unit.
e.g. 1m= 100 cm = 3.28 ft = 39.4 inch
(SI) (CGS) (FPS)
SOLVED EXAMPLE
Ex. The accelaration due to gravity is 9.8 m s–2. Give its value in ft s–2

Sol. As 1m = 3.2 ft ∴ 9.8 m/s2 = 9.8 × 3.28 ft/s2 = 32.14 ft/s2 ≈ 32 ft/s2
Question For Practice
Q1. The value of Gravitational constant G in MKS system is 6.67 × 10–11 N-m2/kg2 . What will be its value in CGS
system ?
Q2. Name the smallest and largest units of length. (A) (B)
(a) Base unit (i) N
(b) Derived unit (ii) hp
Q3. Match the type of unit (column A) (c) Improper unit (iii) kg - wt
with its corresponding example (column B) (d) Practical unit (iv) rad
(e) Supplementary unit (v) kg

Ans. (1) 6.67 × 10–8 cm3/g s2 (2) fermi and parsec (3) (a) -(v), (b) -(i) (c) - (iii), (d) -(ii) (e)-(iv)

DIMENSIONS :
Dimensions of a physical quantity are the powers (or exponents) to which the base quantities are raised to
represent that quantity.
1. Dimensional formula : The dimensional formula of any physical quantity is that expression which represents how
and which of the base quantities are included in that quantity.
It is written by enclosing the symbols for base quantities with appropriate powers in square brackets i.e. [ ]
Ex. Dim. formula of mass is [M1L0 T0 ] and that of speed (= distance/time) is [M0L1T–1]
2. Dimensional equation : The equation obtained by equating a physical quantity with its dimensional formula
is called a dimensional equation. e.g. [v] = [M0L1T–1]
For example [F] = [MLT–2] is a dimensional equation, [MLT–2] is the dimensional formula of the force and the
dimensions of force are 1 in mass, 1 in length and –2 in time

APPLICATIONS OF DIMENSIONAL ANALYSIS :


n2 = numerical value in II system
1. To convert a physical quantity from one system of units to the other :
n1 = numerical value in I system
This is based on a fact that magnitude of a physical quantity
M1 = unit of mass in I system
remains same whatever system is used for measurement i.e.
M2 = unit of mass in II system
magnitude = numeric value (n) × unit (u) = constant or n1u1 = n2u2
L1 = unit of length in I system
So if a quantity is represented by [M a L b T c ] then L2 = unit of length in II system
a b c
T1 = unit of time in I system
u M  L  T  T2 = unit of time in II system
n2 = n1 1 = n1  1   1   1 
u2  M2   L 2   T2 
2. To check the dimensional correctness of a given physical relation :
If in a given relation, the terms on both the sides have the same dimensions, then the relation is
dimensionally correct. This is known as the principle of homogeneity of dimensions.
3. To derive relationship between different physical quantities :
Using the same principle of homogeneity of dimensions new relations among physical quantities can be
derived if the dependent quantities are known.

14 E
ALLEN Basic Mathematics
Limitations of this method :
(i) This method can be used only if the dependency is of multiplication type. The formula containing
exponential, trignometrical and logarithmic functions can't be derived using this method. Formulae
containing more than one term which are added or subtracted like s = ut +at2 /2 also can't be
derived.
(ii) The relation derived from this method gives no information about the dimensionless constants.
(iii) In a given equation correctness of numerical value or dimensionless function can not be checked.
(iv) We cannot derive an equation which depends more than three physical quantities.
(v) We cannot find out wheather physical quantity is scalar or a vector.
SOLVED EXAMPLE
Ex. Convert 1 newton (SI unit of force) into dyne (CGS unit of force)
Sol. The dimensional equation of force is [F] = [M1 L1 T–2]
Therefore if n1, u1, and n2, u2 corresponds to SI & CGS units respectively, then
1 1 −2 −2
 M1   L 1   T1   kg   m   s 
n2 = n1 
M   L   T  = 1  g   cm   s  = 1 × 1000 × 100 × = 105 ∴ 1 newton = 105 dyne.
 2  2  2    

L
Ex. Check the accuracy of the relation T = 2π for a simple pendulum using dimensional analysis.
g
Sol. The dimensions of LHS = the dimension of T = [M0 L0 T1]
12
 dimensions of length 
The dimensions of RHS = 
 dimensions of acceleration 
(Q 2π is a dimensionless constant)

= (L/LT–2)1/2 = (T2 )1 2 = [T] =[ M0 L0 T1]


Since the dimensions are same on both the sides, the relation is correct.
Ex. Find relationship between speed of sound in a medium (v), the elastic constant (E) and the density of the medium
(ρ).
Sol. Let the speed depends upon elastic constant & density according to the relation
v ∝ Eaρb or v = KEaρb K = a dimensionless constant of proportionality
Considering dimensions of the quantities [v] = M0 L T–1
[force] [M1L1 T −2 ]
[stress] [area] [L2 ]
= = = [M1L−1 T −2 ]
[E] = [strain] [∆l] [L1 ] ∴ [Ea] = [Ma L–a T–2a]
[l ] [L1 ]
Q [ρ] = [mass]/[volume] = [M]/[L3] = [M1 L–3 T0] ∴ [ρb] = [Mb L–3b T0]
Equating the dimensions of the LHS and RHS quantities of equation (1), we get
[M0 L1 T–1] = [Ma L–a T–2a] [Mb L–3b T0] or [M0 L1 T–1] = [Ma+b L–a–3b T–2a]
∴ a+b =0 , – a – 3b = 1 and –2a = –1
1 1 E
On solving a = ,b= − so the required relation is v = K
2 2 ρ
Question For Practice
Q1. Match the following :
(i) Dimensional variable (a) π
(ii) Dimensionless variable (b) Force
(iii) Dimensional constant (c) Angle
(iv) Dimensionless constant (d) Gravitational constant
Q2. Find the dimensions of the following quantities :
(a) Temperature (b) Kinetic energy (c) Pressure (d) Angular speed
Q3. Find the dimensions of Planck's constant (h).
Q4. Centripetal force (F) on a body of mass (m) moving with uniform speed (v) in a circle of radius (r) depends upon
m, v and r. Derive a formula for the centripetal force using theory of dimensions.
Ans. 1. (i) -(b) (ii) -(c) (iii) -(d) (iv) -(a) 2.(a) [M0 L0 T0K1] (b) [M L2 T–2] (c) [M L–1 T–2] (d) [M0 L0 T–1]
2
mv
3. [M L2 T–1] 4. F = K
r

E 15
JEE ( Main) - Physics ALLEN
SIGNIFICANT FIGURES & ERROR IN MEASUREMENTS
SIGNIFICANT FIGURES :
The significant figures (SF) in a measurement are the figures or digits that are known with certainity plus one
that is uncertain.
Significant figures in a measured value of a physical quantity tell the number of digits in which we have
confidence. Larger the number of significant figures obtained in a measurement, greater is its accuracy and vice
versa.
Rules to find out the number of significant figures :
I Rule : All the non-zero digits are significant e.g. 1984 has 4 SF.
II Rule : All the zeros between two non-zero digits are significant. e.g. 10806 has 5 SF.
III Rule : All the zeros to the left of first non-zero digit are not significant. e.g.00108 has 3 SF.
IV Rule : If the number is less than 1, zeros on the right of the decimal point but to the left of the first
non-zero digit are not significant. e.g. 0.002308 has 4 SF.
V Rule : The trailing zeros (zeros to the right of the last non-zero digit) in a number with a decimal point are
significant. e.g. 01.080 has 4 SF.
VI Rule : The trailing zeros in a number without a decimal point are not significant e.g. 010100 has 3 SF.
But if the number comes from some actual measurement then the trailing zeros become significant.
e.g. m = 100 kg has 3 SF.
VII Rule : When the number is expressed in exponential form, the exponential term does not affect the
number of S.F. For example in x = 12.3 = 1.23 × 101 = .123 × 102 = 0.0123 × 103 = 123 × 10 – 1
each term has 3 SF only.
ROUNDING OFF
To represent the result of any computation containing more than one uncertain digit, it is rounded off to
appropriate number of significant figures.
Rules for rounding off the numbers :
I Rule : If the digit to be rounded off is more than 5, then the preceding digit is increased by one. e.g. 6.87≈ 6.9
II Rule : If the digit to be rounded off is less than 5, than the preceding digit is unaffected and is left
unchanged. e.g. 3.94 ≈ 3.9
III Rule : If the digit to be rounded off is 5 then the preceding digit is increased by one if it is odd and is left
unchanged if it is even. e.g. 14.35 ≈ 14.4 and 14.45 ≈ 14.4
RULES FOR ARITHMETICAL OPERATIONS WITH SIGNIFICANT FIGURES :
I Rule : In addition or subtraction the number of decimal places in the result should be equal to the number of
decimal places of that term in the operation which contain lesser number of decimal places.
e.g. 12.587 – 12.5 = 0.087 = 0.1 (Q second term contain lesser i.e. one decimal place)
II Rule : In multiplication or division, the number of SF in the product or quotient is same as the smallest
number of SF in any of the factors. e.g. 5.0 × 0.125 = 0.625 = 0.62
GOLDEN KEY POINTS
• To avoid the confusion regarding the trailing zeros of the numbers without the decimal point the best way
is to report every measurement in scientific notation (in the power of 10). In this notation every number is
expressed in the form a × 10b , where a is the base number between 1 and 10 and b is any positive or
negative exponent of 10. The base number (a) is written in decimal form with the decimal after the first
digit. While counting the number of SF only base number is considered (Rule VII).
• The change in the unit of measurement of a quantity does not affect the number of SF. For example in
2.308 cm = 23.08 mm = 0.02308 m = 23080 µm each term has 4 SF.

SOLVED EXAMPLE
Ex. Write down the number of significant figures in the following.
(a) 165 3SF (see rule I) (b) 2.05 3 SF (see rules I & II)
(c) 34.000 m 5 SF (see rules I & V) (d) 0.005 1 SF (see rules I & IV)
(e) 0.02340 N m–1 4 SF (see rules I, IV & V) (f) 26900 3 SF (see rule VI)
(g) 26900 kg 5 SF (see rule VI)

16 E
ALLEN Basic Mathematics
Ex. The length, breadth and thickness of a metal sheet are 4.234 m, 1.005 m and 2.01 cm respectively. Give the
area and volume of the sheet to correct number of significant figures.
Sol. length (l) = 4.234 m breadth (b) = 1.005 m
thickness (t) = 2.01 cm = 2.01 × 10–2 m
Therefore area of the sheet =2(l× b + b× t + t× l)
= 2 ( 4.234 × 1.005 + 1.005 × 0.0201 + 0.0201 × 4.234) m2
= 2 ( 4.3604739) m2 = 8.720978 m2
Since area can contain a maxm of 3 SF (Rule II of article 4.2) therefore, rounding off, we get Area = 8.72 m2
Like wise volume = l × b × t = 4.234 × 1.005 × 0.0201 m3 = 0.0855289 m3
Since volume can contain 3 SF, therefore, rounding off, we get Volume = 0.0855 m3
Question For Practice
Q 1. Write the following in scientific notation :
(a) 3256 g (b) .0010 g (c) 50000 g (5 SF) (d) 0.3204
Q 2. Give the number of significant figures in the following :
(a) 0.165 (b) 4.0026 (c) 0.0256 (d) 165
4 23
(e) 0.050 (f) 2.653 × 10 (g) 6.02 × 10 (h) 0.0006032
Q 3. Calculate area enclosed by a circle of diameter 1.06 m to correct number of significant figures.
Q 4. Subtract 2.5 × 104 from 3.9 × 105 and give the answer to correct number of significant figures.
Q 5. The mass of a box measured by a grocer's balance is 2.3 kg. Two gold pieces of masses 20.15 g and 20.17 g
are added to the box. What is (a) total mass of the box (b) the difference in masses of gold pieces to correct
significant figures.
Ans. 1. (a) 3.256 × 103 g (b) 1.0 × 10–3 g (c) 5.0 × 104 g (d) 3.204 × 10-1
2. (a) 3 (b) 5 (c) 3 (d) 3 (e) 2 (f) 4 (g) 3 (h) 4
2
3. 0.882 m (3 SF)
4. 3.6 × 105 5. (a) Total mass = 2.3 kg (b) Difference in masses = 0.02g

ERRORS
Errors can be expressed in the following ways :-
Absolute Error (∆a) : The difference between the true value and the individual measured value of the
quantity is called the absolute error of the measurement.
Suppose a physical quantity is measured n times and the measured values are a1, a2, a3 ..........an. The arithmetic
a1 + a2 + a 3 + ..............a n 1 n
mean (am) of these values is am =
n
= ∑ ai
n i =1
.........(1)

If the true value of the quantity is not given then mean value (am) can be taken as the true value. Then the
absolute errors in the individual measured values are –
∆a1 = am – a1
∆a2 = am – a2
......................
......................
∆an = am – an
The arithmetic mean of all the absolute errors is defined as the final or mean absolute error (∆a)m or ∆a of the

|∆a1 |+|∆a2 |+...........+|∆a n | 1 n


value of the physical quantity a (∆a)m = = ∑|∆a i | .........(2)
n n i =1
So if the measured value of a quantity be 'a' and the error in measurement be ∆a, then the true value (at) can be
written as at = a ± ∆a .........(3)
Relative or Fractional Error : It is defined as the ratio of the mean absolute error (( ∆a)m or ∆a) to the true
value or the mean value (am or a ) of the quantity measured.
Mean absolute error ( ∆a)m ∆a
Relative or fractional error = = or .........(4)
Mean value am a

E 17
JEE ( Main) - Physics ALLEN
When the relative error is expressed in percentage, it is known as percentage error,
percentage error = relative error × 100 %
mean absolute error ∆a
or percentage error = × 100% = × 100% .........(5)
true value a
PROPAGATION OF ERRORS IN MATHEMATICAL OPERATIONS :
(a) If x = a + b, then the maximum possible absolute error in measurements of x will be ∆x = ∆a + ∆b
(b) If x = a – b, then the maximum possible absolute error in measurement of x will be ∆x = ∆a + ∆b
a ∆x ∆a ∆b
(c) If x = then the maximum possible fractional error will be = +
b x a b
∆x ∆a
(d) If x = an then the maximum possible fractional error will be = n
x a
a n bm ∆x ∆a ∆b ∆c
(e) If x = p then the maximum possible fractional error will be = n + m + p
c x a b c
∆x 1 ∆a
(f) If x = logea then the maximum possible fractional error will be =
x x a
GOLDEN KEY POINTS
• Systematic errors are repeated consistently with the repetition of the experiment and are produced due to
improper conditions or procedures that are consistent in action whereas random errors are accidental and
their magnitude and sign cannot be predicted from the knowledge of the measuring system and conditions of
measurement.
Systematic errors can therefore be minimised by improving experimental techniques, selecting better instru-
ments and improving personal skills whereas random errors can be minimised by repeating the observation
several times.
• Mean absolute error has the units and dimensions of the quantity itself whereas fractional or relative error is
unitless and dimensionless.
• Absolute errors may be positive in certain cases and negative in other cases.

SOLVED EXAMPLE

Ex. Following observations were taken with a vernier callipers while measuring the length of a cylinder.
3.29 cm, 3.28 cm, 3.29 cm, 3.31 cm,
3.28 cm, 3.27 cm, 3.29 cm, 3.30 cm
Then find
(a) Most accurate length of the cylinder. (b) Absolute error in each observation.
(c) Mean absolute error (d) Relative error (e) Percentage error
Express the result in terms of absolute error and percentage error.

Sol. (a) Most accurate length of the cylinder will be the mean length ( l )

3.29 + 3.28 + 3.29 + 3.31 + 3.28 + 3.27 + 3.29 + 3.30


l= = 3.28875 cm or l = 3.29 cm
8
(b) Absolute error in the first reading = 3.29 – 3.29 = 0.00 cm
Absolute error in the second reading = 3.29 – 3.28 = 0.01 cm
Absolute error in the third reading = 3.29 – 3.29 = 0.00 cm
Absolute error in the forth reading = 3.39 – 3.31 = –0.02 cm
Absolute error in the fifth reading = 3.29 – 3.28 = 0.01 cm
Absolute error in the sixth reading = 3.29 – 3.27 = 0.02 cm
Absolute error in the seventh reading = 3.29 – 3.29 = 0.00 cm
Absolute error in the last reading = 3.29 – 3.30 = –0.01 cm

18 E
ALLEN Basic Mathematics
0.00 + 0.01 + 0.00 + 0.02 + 0.01 + 0.02 + 0.00 + 0.01
(c) Mean absolute error = ∆l = = 0.01 cm
8

∆l 0.01
(d) Relative error in length = = = 0.0030395 = 0.003
l 3.29

∆l
(e) Percentage error = × 100 = 0.003 × 100 = 0.3%
l

So length l = 3.29 cm ± 0.01 cm (in terms of absolute error )

or l = 3.29 cm ± 0.30% (in terms percentage error )

Ex. The inital and final temperatures of water as recorded by an observer are (40.6 ± 0.2)°C and
(78.3 ± 0.3)°C. Calculate the rise in temperature with proper error limits.

Sol. Given θ1 = (40.6 ± 0.2)°C and θ2 = (78.3 ± 0.3)°C

Rise in temp. θ = θ2 – θ1 = 78.3 – 40.6 = 37.7°C. ∆θ = ± (∆θ1 + ∆θ2) = ± (0.2 + 0.3) = ± 0.5°C

∴ rise in temperature = (37.7 ± 0.5)°C

Ex. The length and breadth of a rectangle are (5.7 ± 0.1) cm and (3.4 ± 0.2) cm. Calculate area of the
rectangle with error limits.

Sol. Given l = (5.7 ± 0.1) cm and b = (3.4 ± 0.2) cm

Area A = l × b = 5.7 × 3.4 = 19.38 cm2

∆A  ∆l ∆b   0.1 0.2   0.34 + 1.14  1.48


= ± +  = ±  +  = ±   = ±
A  l b 5.7 3.4 5.7 × 3.4 19.38

1.48 1.48
or ∆A = ± × A=± × 19.8 = ±1.48 ∴ Area = (19.38 ± 1.48) sq. cm
19.38 19.38

Ex. A body travels uniformly a distance (13.8 ± 0.2) m in a time (4.0 ± 0.3) s. Calculate its velocity with error
limits. What is the percentage error in velocity ?

Sol. Given distance s = (13.8 ± 0.2) m and time t = (4.0 ± 0.3) s

s 13.8
velocity v = = = 3.45 ms–1 = 3.5 ms–1
t 4.0

∆v  ∆s ∆t   0.2 0.3   0.8 + 4.14  4.49


=± + =± + =± = ± = ±0.0895
v  s t   13.8 4.0   13.8 × 4.0  13.8 × 4.0

or ∆v = ± 0.0895 × v = ± 0.0895 × 3.45 = ± 0.3087 = ± 0.31

∴ v = (3.5 ± 0.31) ms–1

∆v
Percentage error in velocity = × 100 = ± 0.0895 × 100 = ± 8.95% = ± 9%
v

E 19
JEE ( Main) - Physics ALLEN
Ex. A thin copper wire of length L increase in length by 2% when heated from T1 to T2. If a copper cube having
side 10 L is heated from T1 to T2 what will be the percentage change in

(i) area of one face of the cube and.

(ii) volume of the cube.

Sol. (i) Area A = 10 L × 10 L = 100 L2.

∆A ∆L
Percentage change in area = × 100 = 2 × × 100 = 2 × 2% = 4%
A L

 ∆A ∆100 ∆L ∆L ∆L 
Q = +2 = 0+2 =2  Note :- Constants do not have any error in them.
A 100 L L L 

(ii) Volume V = 10 L × 10 L × 10 L = 1000 L3

∆V ∆L
percentage change in volume = × 100 = 3 = 3 × 2% = 6%
V L

Conclusion :- The maximum percentage change will be observed in volume, lesser in area and the least
(minimum) change will be observed in length or radius.

Question For Practice

Q1. Two rods have lengths measured as (1.8 ± 0.2)m and (2.3 ± 0.1)m. Calculate their combined length with
error limits.

Q2. The original length of wire is (153.7 ± 0.6) cm . It is stretched to (155.3 ± 0.2) cm. Calculate the
elongation in the wire with error limits.

Q3. In an experiment, values of two resistances are measured to be r 1 = (5.0 ± 0.2) ohm and
r2 = (10.0 ± 0.1) ohm. Find the values of total resistance in series with limits of percentage error..

Q4. The radius of a sphere is measured to be (2.1 ± 0.5) cm. Calculate its surface area with absolute error
limits.

Q5. A physical quantity x is calculated from the relation x = a3b2 cd . Calculate percentage error in x, if a, b, c
and d are measured respectively with an error of 1%, 3%, 4% and 2%.

Ans. 1. (4.1 ± 0.3) m 2. (1.6 ± 0.8) cm 3. Rs = 15 ohm ± 2%

4. (55.4 ± 26.4) cm2 5. ± 12%

20 E
ALLEN Basic Mathematics

VECTORS
SCALAR QUANTITIES :
A physical quantity which can be described completely by its magnitude only and does not require a direction
is known as a scalar quantity.
It obey the ordinary rules of algebra.
Ex : Distance, mass, time, speed, density, volume, temperature, current etc.
VECTOR QUANTITIES :
A physical quantity which requires magnitude and a particular direction, when it is expressed.
Ex. : Displacement, velocity, acceleration, force etc.

Vector quantities must obey the rules of vactor algebra.


A vector is represented by a line headed with an arrow. Its length is proportional to its magnitude.
→ → →
A is a vector and A = PQ
→ →
Magnitude of A =|A| or A = PQ
Polar Vector :
Vectors which have initial point or a point of application are called polar vectors.
Ex. : Displacement, force etc.
Axial Vector :
These vectors are used in rotational motion to define rotational effects. Direction
of these vectors is always along the axis of rotation in accordance with right hand
screw rule or right hand thumb rule.
→ → →
Ex. : Small angular displacement ( dθ ), Angular velocity (ω ) , Angular momentum ( J) ,
→ →
Angular acceleration (ω ) and Torque (τ )
Equal Vectors :
Vectors which have equal magnitude and same direction are called equal vectors.
→ →
A= B
Opposite (or Negative) Vectors :
Vectors which have equal magnitude but opposite direction are called opposite vectors.
→ →
AB and BA are opposite vectors
→ →
AB = − BA
Coplanar Vectors :
Vectors located in the same plane are called coplanar vectors.
Note :- Two vectors are always coplanar.
Collinear vectors :
Two vectors acting along same straight line or along parallel straight lines in same
diection or in oposite direction figure shows.
Null or Zero Vector :
A vector having zero magnitude is called null vector.
→ → → → →
Ex :- Sum of two vectors is always a vectorm so, ( A ) + ( − A ) = 0 [Note : (A) + ( − A) ≠ 0 ]

0 is a zero vector or null vector.. c
Co-initial vector :
b
Co-initial vectors are those vectors which have the same initial point.
a
→ → →
a, b and c are co-initial vectors.

E 21
JEE ( Main) - Physics ALLEN
Unit Vectors :
A vector having unit magnitude is called unit vector. It is used to specify direction. A unit vector is represented
by a$ (a cap or a hat or a caret).

→ A Vector → → →
Unit vector in the direction of A is â = → (unit vector = ) A = Aa$ or A =|A|a$
Magnitude of the vector
|A|

A unit vector is used to specify the direction of a vector. Y

Base Vectors

j ∧

In an XYZ co-ordinate frame there are three unit vectors $i , $j and k$ , these are i X

used to indicate X, Y and Z directions respectively. These three unit vectors are k
mutually perpendicular to each other. Z
ADDITION OF TWO VECTORS :
Vector addition can be performed by using following methods (i) Graphical methods (ii) Analytical methods
Addition of two vectors is quite different from simple algebraic sum of two numbers.
Triangle Law of Addition of Two Vectors
If two vectors are represented by two sides of a triangle in same order then their sum or 'resultant vector' is
given by the third side of the triangle taken in opposite order of the first two vectors.


(i) Shift one vector ( B ), without changing its direction, such that its tail coincide with head of the other

vector ( A ).
→ →
(ii) Now complete the triangle by drawing third side, directed from tail of A to head of B
→ →
(it is in opposite order of A and B vectors).
→ → →
(iii) Sum of two vectors is also called resultant vector of these two vectors. Resultant (R ) = A + B
→ → →
Length of R is the magnitude of vector sum i.e. |A + B|
→ → →
Bsin θ

R
∴|R|= |A + B|= A2 + B2 + 2AB cos θ B
α θ
→ →
Let direction of R makes angle α with A A Bcos θ

B sin θ
tan α =
A + B cos θ
D C
PARALLELOGRAM LAW OF ADDITION OF TWO VECTORS :
B
If two vectors are represented by two adjacent sides of a parallelogram which B A+
R=
are directed away from their common point then their sum (i.e. resultant β
vector) is given by the diagonal of the parallelogram passing away through α
A θ
that common point. B
A
→ → → → → → →
AB + AD = AC = R or A+ B = R ⇒ R= A 2 + B2 + 2AB cos θ
B sin θ A sin θ
tan α = and tan β =
A + B cos θ B + A cos θ
22 E
ALLEN Basic Mathematics
SOLVED EXAMPLE
Ex. Two forces of magnitudes 3N and 4N respectively are acting on a body. Calculate the resultant force if the
angle between them is : (i) 0° (ii) 180° (iii) 90°
Sol. (i) θ = 0°, both the forces are parallel, R = A + B
∴ Net force or resultant force R = 3 + 4 = 7N
Direction of resultant is along both the forces
(ii) θ = 180°, both the forces are antiparallel,R = A ~ B
∴ Net force or resultant force R = 4 – 3 = 1N
Direction of net force is along bigger force means along 4N.
(iii) θ = 90°, both the forces are perpendicular
then R= A 2 + B2 + 2AB cos 90o

= A 2 + B2 = 32 + 42 = 5N
3  3
tan α = or α = tan −1   = 37o
4  4
magnitude of resultant is 5N which is acting at an angle of 37° from 4N force.
Ex. Two vectors having equal magnitude of 5 units, have an angle of 60° between them. Find the magnitude of
their resultant vector and its angle from one of the vectors.
Sol. a = 5 unit and θ = 60°
R = a 2 + a 2 + 2a.a cos 60o = a 3 = 5 3 unit
3
a sin 60o 1
2 = tan 30o ∴ α = 30
o
tan α = = =
a + a cos 60o
3 3
2
→ →
Ex. A vector A and B make angles of 20° and 110° respectively with the X-axis. The magnitudes of these
vectors are 5m and 12m respectively. Find their resultant vector.
R
→ →
Sol. angle between the A and B = 110°– 20° = 90° B

R= A + B + 2AB cos 90
2 2 o
= 5 + 12 = 13m
2 2

→ → ° A
Let angle of R from A is α 110 20°
X-axis
B sin θ 12 sin 90o 12 × 1 12
tan α = = = =
A + B cos θ 5 + 12 cos 90o 5 + 12 × 0 5

12 →
or α = tan −1 ( ) with vector A or (α + 20°) with X-axis
5
→ → →
Ex. Figure shows a parallelogram ABCD. Prove that AC + BD = 2 BC
→ → → → → → D C
Sol. AC = AB+ BC BD = BC+ CD [applying triangle law of vectors]
→ → → → → → → → →
Now AC + BD = AB + BC + BC + CD = AB + 2 BC + CD
→ → → → → → → →
But CD = − AB ∴ AC + BD = AB + 2 BC − AB = 2 BC A B
Ex. Two forces each numerically equal to 10 dynes are acting as shown in the figure, then find resultant of these
two vectors.
e
yn

Sol. The angle θ between the two vectors is 120° and not 60°.
d
10

∴ ( )(10)(cos 120o ) = 100 + 100 − 100 = 10 dyne


R = (10)2 + (10)2 + 210 60° 10 dyne

E 23
JEE ( Main) - Physics ALLEN
GOLDEN KEY POINTS
→ → → →
• Vector addition is commutative A+ B = B+ A
→ → → → → →
• Vector addition is associative A+ (B+ C) = (A+ B) + C
• Resultant of two vectors will be maximum when they are parallel i.e. angle between them is zero.
→ → → → → →
R =| A + B | = A 2
+ B 2
+ 2AB cos 0° = (A + B) 2
= (A + B) or | A + B | =| A |+ | B|
max max max
• Resultant of two vectors will be minimum when they are antiparallel angle between them is 180°.
→ →
=| A + B | = A + B + 2AB cos180 = (A − B) = A ~ B (Bigger — smaller)
2 2 o 2
R
min
→ → → → → →
or R =| A |~| B | or | A + B | = | A |~| B|
min min
• Resultant of two vectors of unequal magnitude can never be zero.
• If vectors are of unequal magnitude then minimum three coplanar vectors are required for zero resultant.
• Resultant of two vectors of equal magnitude will be at their bisector.
→ → → →
If |A|=|B| But if |A|>|B| then angle β > α

B
B
B

A+
A+

B R=
θ β
R=

α
θ A A


R will incline more towards the vector
of bigger magnitude.

→ → |B

θ/2
|=
• If two vectors have equal magnitude i.e. |A|=|B| = a and angle a

os
c
between them is θ then resultant will be at the bisector of
2a
R=
→ → θ → → → θ
θ/2
A and B and its magnitude is equal to 2acos θ/2
|R|=|A + B| = 2acos |A|=a
2 2

120o → → → → →
Special Case : If θ = 120° then R = 2a cos = a i.e. If θ = 120° then |R|=|A + B|=|A|=|B|= a
2
• If resultant of two unit vectors is another unit vector then the angle between them (θ) = 120°.
or If the angle between two unit vectors (θ) = 120°, then their resultant is another unit vector.

Addition of More Than Two Vectors (Law of Polygon)


C D
If some vectors are represented by sides of a C
polygon in same order, then their resultant vector B
is represented by the closing side of polygon in the A R

→ → → → → A
D
opposite order. R = A + B + C + D B

GOLDEN KEY POINTS


• In a polygon if all the vectors are in same order then their resultant is a null vector. C
D
→ → → → → →
A+ B+ C+ D+ E = 0
• If n vectors of equal magnitude are arranged at equal angles of separation then their E B
resultant is always zero. A

24 E
ALLEN Basic Mathematics
SUBTRACTION OF TWO VECTORS
B
→ → → →
Let A and B are two vectors. Their difference i.e. A – B can be treated as

→ → → → → → θ
A
sum of the vector A and vector (– B ). A − B = A + (− B )
α
→ → → →
To subtract B from A , invert the direction of B and add to vector A according ding –B

A-
→ →

B
→ →
to law of triangle. |A − B|= A 2 + B2 − 2AB cos θ , θ the angle between A and B . –B

→ B sin θ
Let angle of difference vector from vector A is α then tanα =
A − B cos θ

(Lami's theorem)
→ → →
If three vectors A , B and C are represented both in magnitude and direction by the sides of a triangle taken
γ
A B C
in the same order, then = = S B
sin α sin β sin γ Q
α
→ →
α = angle between B and C . β = angle between A
C
→ → → →
C and A . γ = angle between A and B β
P
GOLDEN KEY POINTS
→ → → →
• The vector subtraction doesn't follow commutative law i.e. A – B ≠ B – A
→ → → → → →
• The vector subtraction doesn't follow associative law i.e. ( A − B ) − C ≠ A − ( B − C )
→ →
• If two vectors have equal magnitude, i.e. |A|=|B|= a and θ is the angle between them, then
→ → θ
|A − B|= a2 + a2 − 2a cos θ = 2a sin
2

→ → → →
Special case : If θ = 60o then 2a sin θ = a i.e. |A − B|=|A|=|B|= a at θ = 60°
2
• If difference of two unit vectors is another unit vector then the angle between them is 60° or
If two unit vectors are at angle of 60°, then their difference is also a unit vector.
• In physics whenever we want to calculate change in a vector quantity, we have to use vector subtraction. For
r r r
example, change in velocity ( ∆v) = v 2 − v1
• If two vectors are such that their sum and their difference vectors have equal magnitude then angle between
the given vectors (θ) = 90°.
→ → → →
|A + B| = |A − B| or A + B + 2AB cos θ = A + B – 2AB cos θ or cos θ = 0 or θ = 90°
2 2 2 2

→ → → → → →
• If A+ B = A− B then B = 0 ( null vector)

RESOLUTION OF VECTORS INTO RECTANGULAR COMPONENTS


When a vector is splitted into components which are at right angle to each
other then the compenents are called rectangular or orthogonal
components of that vector.
→ →
(i) Let vector a = OA in X - Y plane, makes α angle from X-axis.
Draw perpendiculars AB and AC from A on the X-axis and Y-axis respectively.

E 25
JEE ( Main) - Physics ALLEN
→ →
(ii) The length OB is called projection of OA on X-axis or component of OA along X-axis and is represented

by ax. Similarly OC is the projection of OA on Y-axis and is represented by ay.
→ → → →
According to law of vector addition. a = OA = OB+ OC

Thus a has been resolved into two parts, one along OX and the other along OY, which are mutually
perpendicular.
OB
In ∆OAB, = cos α or OB = OA cos α or ax = a cos α
OA
AB
and = sinα or AB = OA sin α = OC or ay = a cos β ∴
OA
ax = a cos α and ay = a cos β
→ →
If $i and $j denote unit vectors along OX and OY respectively then OB = a cos α $i and OC = a sin α $j
So that according to rule of vector addition
→ → → → →
$ $ a = a cos α $i + a sin α $j
OA = OB + OC or a = a x i + a y j or
RECTANGULAR COMPONENTS OF A VECTOR IN THREE DIMENSIONS :
→ →
(i) Consider a vector a represented by OA , as shown in figure. Consider O as origin and draw a rectangular
parallelopiped with its three edges along the X, Y and Z axes.
r r r r
(ii) Vector a is the diagonal of the parallelopiped whose projections on x, y and z axis are a x , a y and a z respectively.

These are the three rectangular components of a . Y
C
→ → → ay
Using triangle law of vector addition OA = O E + EA
A
→ → →
Using parallelogram law of vector addition OE = (OB+ OD) a

→ → → → O
ax B
X
∴ OA = (O B+ OD) + EA
D az
→ → → → → → E

EA = OC ∴ OA = OB + OD + OC
Z

→ → → → → →
$
Now OA = a , OB = a x $i, OC = a y $j and OD = a z K ∴ a = a x $i + a y $j + a z k$
Also (OA)2 = (OE)2 + (EA)2 But (OE)2 = (OB)2 + (OD)2
2 2 2
and EA = OC ∴ (OA)2 = (OB)2 + (OD)2 + (OC)2 or a2 = a x + a y + a z ....(i)

2 2 2
a = a x + a y + az
Directional Cosines
→ Y
Let a makes angle α with x axis, β with y axis and γ with z axis
ax a
ay
cos α = ax = a cos α
a β
α ax
ay
cos β = ay = a cos β γ X
a Z az

az
cos γ = az = a cos γ
a
cos α, cos β and cos γ are directional cosines of the vector.

26 E
ALLEN Basic Mathematics
Putting the value of ax, ay and az in eq. (i) we get a2 = a2 cos2 α + a2 cos2 β + a2 cos2 γ

or cos2 α + cos2 β + cos2 γ = 1 or ( 1 - sin2 α ) + ( 1 - sin2 β ) + ( 1 - sin2 γ ) = 1


or 3 - (sin2 α + sin2 β + sin2 γ ) = 1 or sin2 α + sin2 β + sin2 γ = 2
GOLDEN KEY POINTS
• A vector can be resolved into maximum infinite number of components.
For example 10$i = $i + $i + $i............10 times
$i $i $i
=
+ + ............20 times
2 2 2
• Maximum number of rectangular components of a vector in a plane is two. But maximum number of rectangular
components in space (3-dimensions) is three which are along X, Y and Z axes.
• A vector is independent of the orientation of axes but the components of that vector depends upon the
orientation of axes.
• The component of a vector along its perpendicular direction is always zero.

SOLVED EXAMPLE

→ →
Ex. If P = 3iˆ + 4ˆj + 12kˆ then find magnitude and the direction cosines of the P .

Sol. magnitude P is = Px + Py + Pz = 32 + 42 + 122 = 169 = 13
2 2 2

Px 3 Py 4 P 12
direction cosines cos α = = , cos β = = , cos γ = z =
P 13 P 13 P 13
Ex. Find out the angle made by ( ˆi + ˆj ) vector from X and Y axes respectively.
Y

i+ ∧
j
= ∧
a = ax + ay = 1 + 1 = 2
2 2 2 2
Sol.

a
ax 1
cos α = =
°
∴ α = 45°
45

a 2 5°
β=

=4
α
ay 1
cos β = = ∴ β = 45°
X
a 2
ˆi + ˆj is at bisector of X and Y axes.

Ex. Find out the angle made by A = ˆi + ˆj + kˆ vector from X, Y and Z axes respectively.
Sol. given Ax = A y = A z = 1
A = Ax + Ay + Az = 1 + 1 + 1 = 3
2 2 2
so

Ax = 1 1 Ay 1 1
cos α = or α = cos −1 cos β = = or β = cos −1
A 3 3 A 3 3
Az 1 1
cos γ = = or γ = cos −1
A 3 3
Ex. A force of 4N is inclined at an angle of 60° from the vertical.
Find out its components along horizontal and vertical directions.
Sol. Vertical Component = 4 cos 60° = 2N
Horizontal component = 4 sin 60° = 2 3 N
Ex. A force is inclined at an angle of 60° from the horizontal. If the horizontal component of the force is 40N,
calculate the vertical component.
Sol. Ax = 40N, Ay = ?, θ = 60°
A
Q Ax = A cosθ ∴ 40 = A cos 60° = or A = 80N
2
A 3 80 3
Now Ay = A sin 60° = = = 40 3 N = 40 × 1.732 = 69.28 N
2 2
E 27
JEE ( Main) - Physics ALLEN
→ →
Ex. Determine that vector which when added to the resultant of P = 2iˆ + 7jˆ − 10kˆ and Q = ˆi + 2ˆj + 3kˆ gives a
unit vector along X-axis.
→ → →
ˆ + (iˆ + 2jˆ + 3k)
Sol. Resultant R = P + Q = (2iˆ + 7jˆ − 10k) ˆ = 3iˆ + 9ˆj − 7kˆ

But R + required vector = î

or ˆ = −2iˆ − 9ˆj + 7kˆ
required vector = î – R = ˆi − (3iˆ + 9ˆj − 7k)

Ex. ABC is an equilateral triangle. Length of each side is 'a' and centroid is point O. Find C

→ → → → → →
(i) AB + BC + CA = ? (ii) OA + OB + OC = ?
a a
→ → →
(iii) If |AB + BC + AC | = n a then n = ?
O
→ → →
(iv) If AB + AC = n AO then n = ? A
a
B
→ → → → → → →
Sol. (i) AB, BC and CA form a closed triangle in the same order. ∴ AB + BC + CA = 0
→ → →
(ii) OA, OB and OC are three vectors of equal magnitude and are separated by 120° each
→ → → →
∴ OA + OB + OC = 0

→ → → → → → → →
(iii) AB + BC = AC ⇒ |AB + BC + AC|=|2 AC|= 2|AC|= 2a ∴ n = 2
→ → → → → → → → → → →
(iv) AB = AO + OB and AC = AO + OC ∴ AB + AC = 2 AO + OB + OC ...(1)
→ → → → → → → →
but OA + OB + OC = 0 ∴ OB + OC = − OA = AO ..(2)
→ → → → → → →
by (1) and (2) AB + AC = 2AO + AO AB + AC = 3AO ∴n=3
→→ →
Ex. Add vectors A, B and C which have equal magnitude of B Y
A

50 unit and are inclined at angles of 45°, 135° and 315°


13

45°
18 X
respectively from x-axis. 0°

→ →
Sol. Angle between B and C =315°-135°= 180° ∴ They balance each other C

so sum these three is A = 50 unit at 45° from X-axis
Ex. The sum of three vectors shown in figure, is zero.
C

(i) What is the magnitude of vector OB ? D 45° B
O

(ii) What is the magnitude of vector OC ?
A = 10N

Sol. Resolve OC into two rectangular components.

OB = OC cos 45o and OA =OC sin 45o

 1 
⇒ OC =
10
sin 450
= 10 2 N (
⇒ OB = 10 2 )   = 10N
2

28 E
ALLEN Basic Mathematics
MULTIPLICATION OR DIVISION OF A VECTOR BY A SCALAR :

→ → r → A
If there is a vector A and a scalar K and if B = KA and C = then
K
(a) In multiplication of a vector by a scalar the magnitude becomes K times while the direction remains
→ →
same. So that angle between A and B is zero.
(b) In division of a vector by a scalar,the magnitude becomes (1/K) times and the direction remains
→ →
same. So that angle between A and C is zero.
GOLDEN KEY POINTS
• A scalar or a vector, can not be divided by a vector.
• Vectors of different types can be multiplied to generate new physical quantities which may be a scalar or a
vector. If, in multiplication of two vectors, the generated physical quantity is a scalar, then their product is
called scalar or dot product and if it is a vector, then their product is called vector or cross product.

SCALAR PRODUCT OF TWO VECTORS :


Definition :
The scalar product (or dot product) of two vectors is defined as the product of their magnitudes with cosine of
the angle between them.
→ →
Thus if there are two vectors A and B having angle θ between them then their scalar product is written as
→ →
A . B = AB cos θ
GOLDEN KEY POINTS

• It is always a scalar, which is positive if angle between the vectors is acute (i.e.θ < 90°) and negative if angle
between them is obtuse (i.e. 90° < θ < 180°).
→ → → →
• It is commutative A . B = B . A

→ → → → → → →
• It is distributive A .( B + C ) = A . B + A . C
B
→→ A.B = AB cos θ
• According to definition A . B = AB cos θ θ
X
 → → A
The angle between the vectors −1
θ = cos 
A.B

 AB 
• Scalar product of two vectors will be maximum when cos θ = max =1, i.e. θ =0°,
→→
i.e, vectors are parallel. (A. B) = AB
max
→ →
• Scalar product of two vectors will be zero when cos θ = 0, i.e. θ =90° (A . B ) =0
if the scalar product of two nonzero vectors is zero then vectors are orthogonal or perpendicular to each
other.

• In case of orthogonal unit vectors $i , $j and k$ $i . $j = $j. k$ = k$ . $i = 1 × 1 × cos 90o = 0


• The scalar product of a vector by itself is termed as self dot product and is given by

→ → → → →
A . A = A A cos 0o = A 2 ⇒ | A| = A . A

• In case of unit vector n$ n$ . n$ = 1 × 1 × cos 0o = 1 so n$ . n$ = $i . $i = $j . $j = k$ . k$ = 1


→ →
• In terms of components ˆ
A . B = (A xˆi + A yˆj + A z k).(B ˆ ˆ ˆ
x i + B y j + B z k) =
A xBx + A yBy + A zBz

E 29
JEE ( Main) - Physics ALLEN
VECTOR PRODUCT OF TWO VECTORS : ∧
C =A× B n
Definition : ∧
The vector product or cross product of two vectors is defined as n
a vector having magnitude equal to the product of their
magnitudes with the sine of angle between them, and its direction
is perpendicular to the plane containing both the vectors A B
θ
according to right hand screw rule or right hand thumb rule.
→ →
If A and B are two vectors, then their vector product i.e.
→ → → → → →
A × B is a vector C defined by C = A × B = AB sin θ nˆ
Right Hand Thumb Rule
→ →
Place the vector A and B tail to tail. Now place stretched fingers and thumb of right hand perpendicular to
→ → →
the plane of A and B such that the fingers are along the vector A . If the fingers are now closed through
→ → → →
smaller angle so as to go towards B , then the thumb gives the direction of A × B i.e. C
Right Hand Screw Rule
A×B
→ → →
The direction of A × B i.e., C is perpendicular to the plane containing vectors
→ →
A and B and owards the advancement of a right handed screw rotated from
→ →
A (first vector) to B (second vector) through the smaller angle between them. Thus,

if a right handed screw whose axis is perpendicular to the plane formed by A and
→ → → A B
B is rotated from A to B through the smaller angle between them, then the θ
→ →
direction of advancement of the screw gives the direction A × B .

EXAMPLES OF VECTOR PRODUCT :


→ → → → → →
(i) Torque τ = r ×F (ii) Angular momentum J = r × p

→ → → → → →
(iii) Linear velocity v = ω × r (iv) Linear accelaration a = α × r

→ →→→ →
Here r is position vector and F , p , ω and α are force, linear momentum, angular velocity and

angular accelaration respectively.


GOLDEN KEY POINTS
• Vector product of two vectors is always a vector perpendicular to the plane containing the two vectors, i.e.,
→ →
orthogonal (perpendicular) to both the vectors A and B
A×B=C
• Vector product of two vectors is not commutative i.e.

→ → → → → → → → B
A× B ≠ B × A But |A × B| = |B × A|= AB sin θ A

→ → → →
Note : A× B = − B × A
B
→ → → →
i.e., in case of vectors A × B and B × A, magnitudes are equal but A
directions are opposite [See the figure]
B × A = –C

30 E
ALLEN Basic Mathematics
• The vector product is distributive when the order of the vectors is strictly maintained, i.e.
→ → → → → → →
A × ( B + C) = A× B + A× C
→ →
• According to definition of vector product of two vectors A × B = A B sin θn$
 → →
So
→ →
i.e. −1 |A × B|
|A × B|= A B sin θ θ = sin → → 
 |A||B| 
 
• The vector product of two vectors will be maximum when sin θ = max. = 1, i.e., θ =90°
→ →
|A × B| = AB sin 90o = AB
max
i.e. vector product is maximum if the vectors are orthogonal (perpendicular).
• The vector product of two non-zero vectors will be zero when sinθ = 0,
→ →
i.e. θ = 0° or 180° and |A × A|= 0
Therefore if the vector product of two non-zero vectors is zero, then the vectors are collinear.
• The self cross product, i.e., product of a vector by itself is a zero vector or a null vector.
→ → →
i.e. A × A = (AA sin 0o ) n$ = 0
r r
• In case of unit vector n$ o
nˆ × nˆ = 1 × 1 × sin 0 nˆ = 0 so that $i × $i = $j × $j = k$ × k$ = 0
• In case of orthogonal unit vectors $i , $j and k$ ; according to right hand thumb rule
$i × $j = k$ , $j × k$ = $i , k$ × $i = $j and $j × $i = −k$ , k$ × $j = − $i , $i × k$ = − $j
$i $j k$
→ → A z = $i (A B − A B ) − $j (A B − A B ) + k$ (A B − A B )
• In terms of components A × B = A x Ay
y z z y x z z x x y y x
Bx By Bz

SOLVED EXAMPLE

Ex. Can scalar product be ever negative ?


Sol. Yes. Scalar product will be negative if θ > 90°.
→ →
Q → →
P . Q = PQ cos θ ∴ When θ > 90° then cosθ is negative and P . Q will be negative.

→ → → → → →
Ex. If |A + B | = |A − B |, then find the angle between A and B .

→ → → → ⇒ A 2 + B2 + 2AB cos θ
Sol. Q |A + B | = |A − B | = A 2 + B2 − 2AB cos θ
⇒A2 + B2 + 2AB cos θ = A2 + B2 – 2AB cos θ ⇒ cos θ = 0 ∴ θ = 90°
→ → → →
Ex. If A = 4$i + nj$ − 2k$ and B = 2$i + 3$j + k$ , then find the value of n so that A ⊥ B .
→ →
Sol. Dot product of two mutually perpendicular vectors is zero A . B = 0

∴ (4$i + nj$ − 2k$ ).(2$i + 3$j + k$ ) = 0 ⇒ (4 × 2) + (n × 3) + (−2 × 1) = 0 ⇒ 3n = – 6 ⇒ n = – 2


r r r
( )
r
Ex. ( ) r
( )
If F = 4iˆ − 10jˆ and r = 5iˆ − 3jˆ , then calculate torque τ = r × F

r r
Sol. Here r = 5iˆ − 3jˆ + 0kˆ and F = 4iˆ − 10jˆ + 0kˆ

ˆi ˆj kˆ
→ → → ˆ −50 + 12) = −38kˆ
∴ τ = r × F = 5 −3 0 = ˆi(0 − 0) − ˆj(0 − 0) + k(
4 −10 0

E 31
JEE ( Main) - Physics ALLEN
Ex. Find a unit vector perpendicular to both the vectors (2$i + 3$j + k$ ) and ($i − $j + 2k$ ) .
→ → $
Sol. Let A = 2$i + 3$j + k$ and B = $i − $j + 2k
→ →
r r A× B
unit vector perpendicular to both A and B is n̂ = → →
|A × B|
ˆi ˆj kˆ
→ →
A× B = 2 3 1 = $i (6 + 1) − $j (4 − 1) + k$ (−2 − 3) = 7iˆ − 3jˆ − 5kˆ
1 −1 2
→ → 1
∴ n̂ = (7iˆ − 3jˆ − 5k)
∴ |A × B| = 72 + (−3)2 + (−5)2 = 83 unit 83

GOLDEN KEY POINTS


→ →
→ → A× B
• Unit vector perpendicular to A as well as B is n̂ = .
→ →
|A × B|
→ → → → → → →
• If A , B and C are coplanar, then → → →
A .( B × C ) = 0 . • Angle between ( A + B ) and ( A × B ) is 90o .

GEOMETRICAL MEANING OF VECTOR PRODUCT OF TWO VECTORS :


→ → → →
(i) Consider two vectors A and B which are represented by OP and OQ and ∠POQ = θ
(ii) Complete the parallelogram OPRQ. Join P with Q. Here OP = A and OQ = B.
Draw QN ⊥ OP.
→ →
(iii) Magnitude of cross product of A and B Q R
= AB sinθ , = (OP) (OQ sin θ)
B
= (OP) (NQ) (Q NQ = OQ sin θ) = base × height
= Area of parallelogram OPRQ
θ
base × height (OP) (NQ) 1 → → P
Area of ∆POQ = = = | A × B| O N A
2 2 2
→ →
∴ Area of parallelogram OPRQ = 2[area of ∆ OPQ] = | A × B|
Formulae to Find Area :
→ → 1 → →
If A and B are two sides of a triangle, then its area = | A × B|
2
→ → → →
If A and B are two adjascent sides of a parallelogram then its area =| A × B|
→ → 1 → →
If A and B are diagonals of a parallelogram then its area = | A × B|
2
SOLVED EXAMPLE
→ →
Ex. The diagonals of a parallelogram are expressed as A = 5iˆ − 4ˆj + 3kˆ and B = 3iˆ − 2ˆj − kˆ .
Calculate the magnitude of area of this parallelogram.
→ →
Sol. When A and B are the diagonals of a parallelegram, then its
ˆi ˆj kˆ
1 → → → → −4 3 5 3 5 −4
Area = | A × B| A× B = 5 −4 3 = i −j +k
2 −2 − 1 3 −1 3 −2
3 −2 −1

= ˆi {( −4)( −1) − (3)( −2)} − ˆj {(5)( −1) − (3)(3)} + kˆ {(5)( −2) − ( −4)(3)} = 10iˆ + 14ˆj + 2kˆ
→ → 1 → → 1
| A × B| = (10)2 + (14)2 + (2)2 = 300 area of parallelogram | A × B| × 10 3 = 5 3
2 2
32 E
ALLEN Basic Mathematics
SOLVED EXAMPLES
r r
Ex.1 There are two vectors A = 3$i + $j and B = $j + 2k$ . For these two vectors –
r r
(a) Find the component of A along B in vector form. m.
r r
(b) If A & B are the adjacent sides of a parallalogram then find the magnitude of its area.
r r
(c) Find a unit vector which is perpendicular to both A & B .

r r F Ar .Br I B$ F Ar .Br I Br LM e3$i + $jj.e$j + 2k$ j OP e$j + 2k$ j


Sol. (a) Component of A along B = GH B JK = GH B JK B = M PP 5 =
1
e$j + 2k$ j
MN 5
Q 5

$i $j k$
r r
(b)
$
Area of the parallelogram = A × B = 3 1 0 = 2$i − 6$j + 3k = b g 2
22 + −6 + 32 = 7 units
0 1 2
r r
r r A ×B 2$i − 6$j + 3k$ 2 6 3$
(c) Unit vector perpendicular to both A & B n$ = r r = = $i − $j + k
A×B 7 7 7 7

Ex.2 If vector (a$ + 2b$ ) is perpendicular to vector (5a$ − 4b$ ) , then find the angle between a$ and b$ .

Sol. Q (a$ + 2b$ ) ⊥ (5a$ − 4b$ ) ∴ (a$ + 2b$ ) · (5a$ − 4b$ ) = 0 ⇒ 5a$ . a$ − 4a$. b$ + 10b$ . a$ − 8 b$ . b$ = 0

1 1
⇒ 5 + 6a$ . b$ – 8 = 0 ⇒ 6a$ . b$ = 3 ⇒ a$ . b$ = ⇒ cos θ = ⇒ θ = 60°
2 2

Ex.3 Vectors 2$i + 2$j − 2k$ , 5$i + yj$ + k$ and − $i + 2$j + 2k$ are coplaner then find the value of y.

ax ay az
r r
Sol.
r
We know that condition of coplaner a × b × c = b x ( ) by bz = 0
cx cy cz

2 2 −2
⇒ 5 y 1 = 0 ⇒ 2(2y – 2) – 2(11) – 2(10 + y) = 0
−1 2 2
⇒ 4y – 4 – 22 – 20 – 2y = 0 ⇒ 2y = 46 ⇒ y = 23
Ex.4 In system called the star system we have 1 star kilogram = 10 20 kg. 1 starmetre = 10 8 m,
1 starsecond = 103 second then calculate the value of 1 joule in this system.
Sol. Dimension of joule = ML2T–2
Value of 1 joule in star system = (10–20) (10–8)2(10–3)–2=10–30star joule

Fv I
Ex.5 The position of a particle at time t is given by the relation x(t) = GH α JK (1 – e
0
–αt) where v0 is a constant and

α > 0. Find the dimensions of v0 and α.


Sol. Dimension of α t = M0L0T0 ⇒ Dimension of α = M0L0T–1
v0
Dimension of = L1 ⇒ Dimension of v0 = M0L1 T–1
α
Ex.6 If velocity, force and acceleration are taken as fundamental quantities instead of mass, length and time
then find the dimension of Young's modulus.
Sol. v = LT–1, F = MLT–2, a = LT–2 & Y = ML–1T–2 ⇒ M = Fa–1, T = va–1, L = v2a–1
⇒ Y = ML–1T–1 = (Fa–1) (v–2a1) (v–2a2) = Fa2v–4

E 33
JEE ( Main) - Physics ALLEN
Ex.7 Write down the values of the following -
(a) $i × $j × k$ (b) $i ⋅ $j + $j ⋅ $j + k$ ⋅ $i

(c) ek$ × $i j ⋅ $i (d)


d
dx
b
log e x g
π/4
(e) cos 120° (f) z sin 2x dx
0
2 3x
de
(g) 2 (h) 2$i − $j − 5k$
dx
d2
(i)
dx 2
e4x 3
− 3x 2 + 2x + 1 j (j)
d
dx
e
cos 4x 2 j
z
+1
1 $i · ( $j × k$ ) + $j · ( k$ × $i ) + k$ ·( $i × $j )
(k) dt (l)
−1
t3

r 1 1
Sol. (a) 0 (b) 1 (c) 0 (d) (e) −
x 2

z LM cos 2x OP = – 1 FG cos 2π − cos 0IJ = − 1 b0 − 1g = 1


π/4 π/4
sin 2x dx = −
N 2 Q 2H K 2
(f)
0
4 0 2

d F I d (3e ) = 3e .3 = 9e
2
(e ) = G e (3x)J =
d 3x d 3x 3x 3x 3x
(g)
dx
2
dx H dx K dx
(h) 2$i − $j − 5k$ = 2 + b −1g + b −5g = 30
2 2 2

d2
(i)
dx 2 e4x 3
− 3x 2 + 2x + 1 = j d
dx
2
(12x – 6x + 2) = 24x – 6

d
e
cos 4x 2 = − sin 4x 2 j
d
4x 2
LM e jOP = –8xsin4x 2
(j)
dx dx N Q
L 1 OP = FG − 1 IJ − FG − 1 I 1 1
z L t OP
+1 +1 +1
−2
t −3
dt = M
MN −2 PQ = M−
N 2t Q GH 2b1g JK GH 2b−1g JJ = − 2 + 2 = 0
(k)
−1 −1
2
−1
2 2
K
(l) $i · ( $j × k$ ) + $j · ( k$ × $i ) + k$ · ( $i × $j ). = $i · $i + $j · $j + k$ · k$ = 1 + 1 + 1 = 3

FG n 2 – n1 IJ
Ex.8 Number of particles is given by n = – D
Hx 2 – x1 K crossing a unit area perpendicular to x–axis in unit

time. Where n1 and n2 are number of particles per unit volume between length (x2 – x1). What is the
dimension of D.
Volume × Length 4
[L ]
Sol. D= = [L2 T–1] =
[L2T]2
[L T ]
Ex.9 If Energy (E), velocity (v) and time (T) are fundamental units. What will be the dimension of
surface tension?
work done Energy LM E OP = L E O Q L = vT
N A Q MN L PQ
E –2 –2
Sol. Surface tension (S) = = = 2 ∴ S= 2 = Ev T
Area Area (vT )
α
Ex.10 Force (F) and density (d) are related as F = then what are the dimension of α and β ?
β+ d

Sol. [α]= [F d ] = [MLT–2] [M1/2 L–3/2] = M3/2 L–1/2 T–2 , [β] = [d]1/2 = [M1L–3T0]1/2 = [M1/2 L–3/2T0]

34 E
ALLEN Basic Mathematics
Ex.11 A famous relation in physics relates moving mass (m) to the rest mass (m0) of a particle in terms of it
speed (v) and the speed of light (c). A student fails to recall the relation correctly and forgets to place c.
m0
He writes m = . Check the dimensional correctness of this reletion and guess where to put the
(1 − v 2 )1 2
missing c.
m0
Sol. (1-v2)1/2 = [=] [M0L0T0]
m
1–v2 = [M0L0T0], v2 = [M0L0T0] , kV = [M0L0T0] , K = [M0L–1T–1]
Then k = velocity of light.
Ex.12 The initial and final temperatures of water in a container are observed as 16 ± 0.6°C and 56 ± 0.3°C.
What is the rise in the temperature of water.
Sol. Rise in temperature = [(56 ± 0.3) – (16 ± 0.6)] = [(56 – 16) ± (0.3 + 0.6)] = 40 ± 0.9°C
Ex.13 Area of a square is (100 ± 2)m2 Determine its side.
Sol. Let side of square be a then area = a2 = (100 ± 2)m2
Q error in a2 is 2% so error in a will be 1%
Hence a = (10 ± 1) m
Ex..14 The maximum percentage errors in the measurement of mass, radius and angular velocity of a ring are
2%, 1% and 1% respectively, then find the maximum percentage errors in the measurement of its –
(a) Moment of inertia about its geometric axis I = MR2.
1 2
(b) Rotational kinetic energy, K =Iω and
2
(c) Angular momentum about geometrical axis J = Iω.
Sol. (a) moment of inertia (I) = MR2
∆I ∆M ∆R
∴ × 100 = × 100 + 2 × 100 = 2% + (2 × 1%) = 4%
I M R
1 2 1
(b) Rotational kinetic energy (K) = Iω = MR2ω2
2 2
∆K ∆M ∆R ∆ω
∴ × 100 = × 100 + 2 × 100 + 2 × 100 = 2% + (2 × 1%) + (2 × 1%) = 6%
K M R ω
∴ Maximum percentage error in the measurement of its kinetic energy = 6%
1
(c) Angular momentum (J) = Iω = MR 2ω
2
∆J ∆M ∆R ∆ω
∴ × 100 = × 100 + 2 × × 100 + × 100 = 2% + (2 × 1%) + 1% = 5%
J M R ω

l
Ex.15 The period of oscillation of a simple pendulum is given by T = 2π where l is about 100cm and is
g
known to have 1mm accuracy. The period is about 2s. The time of 100 oscillations is measured by a
stop watch of least count 0·1 s. Then calculate the percentage error in "g".
l 2
4π l 4π 2l ∆g ∆l 2∆T
Sol. Q T = 2π ⇒ T2 = ⇒g= ⇒ = +
g g T 2 g l T

1mm 0 ⋅1
Here % error in l = × 100 = × 100 = 0·1%
100 cm 100

0 ⋅1
% error in T = × 100 = 0·05 %
2 × 100
∴ % error in g = % error in l + 2 (% error in T) = 0·1 + 2 × 0·05 ⇒ 0·2 %

E 35
JEE ( Main) - Physics ALLEN
NCERT BASED QUESTIONS
1. It x = at + bt2, where x is the distance travelled by the body in kilometre while t is the time in seconds, then find
the unit of b ?

2. Fill in the blanks


(a) The volume of a cube of side 1 cm is equal to .....m3
(b) The surface area of a solid cylinder of radius 2.0 cm and height 10.0 cm is equal to ...(mm)2
(c) A vehicle moving with a speed of 18 km h–1 covers....m in 1 s
(d) The relative density of lead is 11.3. Its density is ....g cm–3 or ....kg m–3.
3. Fill in the blanks by suitable conversion of units
(a) 1 kg m2 s–2 = ....g cm2 s–2
(b) 1 m = ..... ly
(c) 3.0 m s–2 = .... km h–2
(d) G = 6.67 × 10–11 N m2 (kg)–2 = .... (cm)3 s–2 g–1.
4. A calorie is a unit of heat of energy and it equals about 4.2 J where 1J = 1 kg m2 s–2. Suppose we employ a
system of units in which the unit of mass equals α kg, the unit of length equals β m, the unit of time is γ s. Show
that a calorie has a magnitude 4.2 α–1β–2 γ2 in terms of the new units.
5. State the number of significant figures in the following :
(a) 0.007 m2 (b) 2.64 × 1024 kg (c) 0.2370 g cm–3
–2
(d) 6.320 J (e) 6.032 N m (f) 0.0006032 m2
6. The length, breadth and thickness of a rectangular sheet of metal are 4.234 m, 1.005 m, and 2.01 cm
respectively. Give the area and volume of the sheet to correct significant figures.
7. The mass of a box measured by a grocer's balance is 2.300 kg. Two gold pieces of masses 20.15 g and
20.17 g are added to the box. What is (a) the total mass of the box, (b) the difference in the masses of the pieces
to correct significant figures ?
8. A physical quantity P is related to four observables a, b, c and d as follows :

P = a3 b2/ c d

The percentage errors of measurement in a, b, c and d are 1%, 3%, 4% and 2%, respectively. What is the
percentage error in the quantity P ? If the value of P calculated using the above relation turns out to be 3.763,
to what value should you round off the result ?
9. A book with many printing errors contains four different formulas for the displacement y of a particle undergo-
ing a certain periodic motion :
(a) y = a sin 2πt/T (b) y = a sin υt

(c) y = (a/T) sin t/a (d) y = (a 2 ) (sin 2πt /T + cos 2πt /T)
(a = maximum displacement of the particle, υ = speed of the particle. T = time-period of motion). Rule out the
wrong formulas on dimensional grounds.
10. A famous relation in physics relates ‘moving mass’ m to the .rest mass. mo of a particle in terms of its speed υ
and the speed of light, c. (This relation first arose as a consequence of special relativity due to Albert Einstein).
A boy recalls the relation almost correctly but forgets where to put the constant c. He writes :
m0
m=
(1 − υ2 )1/ 2
Guess where to put the missing c.
36 E
ALLEN Basic Mathematics

11. What is the magnitude of the component of 9iˆ − 9ˆj + 19kˆ vector along x-axis ?

12. Can displacement vector be added to force vector?

13. Why vectors cannot be added algebraically ?

14. How will you prove that the two vectors are parallel ?

15. How will you prove that two vector are perpendicular?

16. State, for each of the following physical quantities, if it is a scalar or a vector : volume, mass, speed,
acceleration, density, number of moles, velocity, angular frequency, displacement, angular velocity.

17. Pick out the two scalar quantities in the following list :

force, angular momentum, work, current, linear momentum, electric field, average velocity, magnetic
moment, relative velocity.

18. Pick out the only vector quantity in the following list :

Temperature, pressure, impulse, time, power, total path length, energy, gravitational potential, coefficient
of friction, charge.

19. State with reasons, whether the following algebraic operations with scalar and vector physical quantities
are meaningful :

(a) adding any two scalars, (b) adding a scalar to a vector of the same dimensions , (c) multiplying any
vector by any scalar, (d) multiplying any two scalars, (e) adding any two vectors, (f) adding a component
of a vector to the same vector.

20. Read each statements below carefully and state with reasons, if it is true or false : (a) The magnitude of
a vector is always a scalar, (b) each component of a vector is always a scalar, (c) the total path length is
always equal to the magnitude of the displacement vector of a particle. (d) the average speed of a particle
(defined as total path length divided by the time taken to cover the path) is either greater or equal to the
magnitude of average velocity of the particle over the same interval of time, (e) Three vectors not lying
in a plane can never add up to give a null vector.

E 37
JEE ( Main) - Physics ALLEN
SOLUTION
1. [x] = [bt2]

[x]
⇒b= [ t2 ]
∴ unit of b = km/s2

1
2. (a) 1 cm = m
100
Volume of the cube = 1 cm3

 1   1   1 
But 1 cm3 = 1 cm × 1 cm × 1 cm =   m×  m× m
 100   100   100 
∴ 1 cm3 = 10–6 m3
Hence, the volume of a cube of side 1 cm is equal to 10–6 m3.
(b) The total surface area of a cylinder of radius r and height h is
S = 2πr (r + h).
Given that,
r = 2 cm = 2 × 1 cm = 2 × 10 mm = 20 mm
h = 10 cm = 10 × 10 mm = 100 mm
∴ S = 2 × 3.14 × 20×(20 + 100) = 15072 = 1.5 × 104mm2
(c) Using the conversion,

5
1 km/h = m/s
18
5
18 km/h = 18 × = 5 m/s
18
Therefore, distance can be obtained using the relation :
Distance = Speed × Time = 5 × 1 = 5 m
Hence, the vehicle covers 5 m in 1 s.
(d) Relative density of a substance is given by the relation.

Density of substance
Relative density =
Density of water
Density of water = 1 g/cm3
Density of lead = Relative density of lead × Density of water
= 11.3 × 1 = 11.3 g/cm3
1
Again, 1g = kg.
1000
1 cm3 = 10–6 m3

10 −3
1 g/cm3 = kg/m3 = 103 kg/m3
10 −6
∴ 11.3 g/cm3 = 11.3 × 103 kg/m3
38 E
ALLEN Basic Mathematics
3. (a) 1 kg = 103g
1m2 = 104 cm2
1 kg m2 s–2 = 1 kg × 1m2 × 1 s–2
= 103 g × 104 cm2 × 1s–2 = 107 g cm2 s–2
(b) Light year is the total distance travelled by light in one year.
1 ly = Speed of light × One year
= ( 3 × 108 m/s) × (365 × 24 × 60 × 60 s)
= 9.46 × 1015 m

1
∴1m= = 1.057 × 10–16 ly
9.46 × 1015

(c) 1 m = 10–3 km

1
Again, 1 s = h
3600

1 s–1 = 3600 h–1


1 s–2 = (3600)2 h–2
∴ 3 m s–2 = (3 × 10–3 km) × (3600)2h–2 = 3.88 × 10–4 km h–2
(d) 1 N = 1 kg m s–2
1 kg = 10–3 g–1
1 m3 = 106 cm3
∴ 6.67 × 10–11 N m2 kg–2 = 6.67 × 10–11 × (1 kg m s–2 ) ( 1 m2) (1 s–2)
= 6.67 × 10–11 × (1 kg × 1 m3 × 1 s–2)
= 6.67 × 10–11 × (10–3 g–1) × (106 cm3) × (1 s–2)
= 6.67 × 10–8 cm3 s–2 g–1
4. Given that,
1 caloric = 4.2 (1 kg) (1 m2) (1 s–2)
New unit of mass = α kg
1
Hence, in terms of the new unit, 1 kg = = α–1
α
In terms of the new unit of length,

1
1m= = β–1 or 1 m2 = β–2
β

And, in terms of the new unit of time,


1
1s= = γ–1
γ
1 s2 = γ–2

1s–2 = γ2

∴ 1 calorie = 4.2 (1α–1) (1 β–2) (1 γ2) = 4.2 α–1β–2γ2

E 39
JEE ( Main) - Physics ALLEN
2
5. (a) The given quantity is 0.007 m
If the number is less than one, then all zeros on the right of the decimal point (but left to the first non-zero) are
insignificant. This means that here, two zeros after the decimal are not significant. Hence, only 7 is a significant
figure in this quantity.
(b) 2.64 × 1024 kg
The given quantity is 2.68 × 1024 kg.
Here, the power of 10 is irrelevant for the determination of significant figures. Hence all digits i.e., 2, 6, and 4
are significant figures.
(c) 0.2370 g cm–3
The given quantity is 0.2370 g cm–3.
For a number with decimals, the trailing zeroes are significant. Hence, besides digits 2, 3 and 7, 0 that appears
after the decimal point is also a significant figure.
(d) 6.320 J
The given quantity is 6.320 J
For a number with decimals, the trailing zeroes are significant. Hence, all four digits appearing in the given
quantity are significant figures.
(e) 6.032 N m–2
The given quantity is 6.032 Nm–2 .
All zeroes between two non-zero digits are always significant.
(f) 0.0006032 m2
The given quantity is 0.0006032 m2.
If the number is less than one, than the zeroes on the right of the decimal point (but left to the first non-zero) are
insignificant. Hence, all three zeroes appearing before 6 are not significant figures. All zeros between two non-
zero digits are always significant. Hence, the remaining four digits are significant figures.
6. Length of sheet, l = 4.234 m
Breadth of sheet, b = 1.005 m
Thickness of sheet, h = 2.01 cm = 0.0201 m
The given table lists the respective significant figures :

Significant
Quantity Number
Figure

l 4.234 4

b 1.005 4

h 2.01 3
Hence, area and volume both must have least significant figures i.e., 3.
Surface area of the sheet = 2( l × b + b × h + h × l )
= 2 ( 4.234 × 1.005 + 1.005 × 0.0201 + 0.0201 × 4.234 )
= 2 (4.25517 + 0.02620 + 0.08510)
= 2 × 4.360
= 8.72 m2
Volume of the sheet = l × b × h
= 4.234 × 1.005 × 0.0201
= 0.0855 m3
This number has only 3 significant figures i.e., 8, 5 and 5.

40 E
ALLEN Basic Mathematics
7. Mass of grocer's box = 2.300 kg
Mass of gold piece I = 20.15 g = 0.02015 kg
Mass of gold piece II = 20.17 g = 0.02017 kg
Total mass of the box = 2.3 + 0.02015 + 0.02017 = 2.34032 kg
In addition, the final result should retain as many decimal places as there are in the number with the least
decimal places. Hence, the total mass of the box is 2.3 kg.
Difference in masses = 20.17 – 20.15 = 0.02 g
In subtraction, the final result should retain as many decimal places as there are in the number with the least
decimal places.

a 3 b2
8. P=
cd

∆P 3∆a 2∆b 1 ∆c ∆d
= + + +
P a b 2 c d

 ∆P   ∆a ∆b 1 ∆c ∆d 
 P × 100  % =  3 × a × 100 + 2 × b × 100 + 2 c × 100 + d × 100  %
   

1
= 3 ×1 + 2 ×3 + ×4 + 2
2
= 3 + 6 + 2 + 2 = 13 %
Percentage error in P = 13 %
Value of P is given as 3.763.
By rounding off the given value to the first decimal place, we get P = 3.8.
9. (a) Answer : Correct

2πt
y = a sin
T
Dimension of y = M0 L1 T0
Dimension of a = M0 L1 T0

2πt
Dimension of sin = M0 L0 T0
T

Q Dimension of L.H.S = Dimension of R.H.S


Hence, the given formula is dimensionally correct.
(b) Answer: Incorrect
y = a sin vt
Dimension of y = M0 L1 T0
Dimension of a = M0 L1 T0
Dimension of vt = M0 L1 T–1 × M0 L0 T1 = M0 L1 T0
But the argument of the trigonometric function must be dimensionless, which is not so in the given case.
Hence, the given formula is dimensionally incorrect.

E 41
JEE ( Main) - Physics ALLEN
(c) Answer: Incorrect

a t
y =   sin  
T a
Dimension of y = M0L1T0

a
Dimension of = M0L1T–1
T

t
Dimension of = M0 L–1 T1
a
But the argument of the trigonometric function must be dimensionless, which is not so in the given case.
Hence, the formula is dimensionally incorrect.
(d) Answer: Correct

( 

) t t
y = a 2  sin 2 π + cos 2 π 
T T

Dimension of y = M0 L1 T0
Dimension of a = M0 L1 T0

t
Dimension of = M0 L0 T0
T
Since the argument of the trigonometric function must be dimensionless (which is true in the given case), the
dimensions of y & a are same. Hence it is dimensionally correct.
10. Given the relation,
m0
m= 1

(1 − v ) 2 2

Dimension of m = M1 L0 T0
Dimension of m0= M1 L0 T0
Dimension of v = M0 L1 T–1
Dimension of v2 = M0 L2 T–2
Dimension of c = M0 L1 T–1
The given formula will be dimensionally correct only when the dimension of L.H.S is the same as that of

(
R.H.S. This is only possible when the factor, 1 − v 2 ) 2 is dimensionless i.e., (1–v2) is dimensionless. This is

only possible if v2 is divided by c2. Hence, the correct relation is

m0
m= 1
.
 v  2 2
1 − 2 
 c 

11. 9

12. No.

42 E
ALLEN Basic Mathematics
13. Because they possess both magnitude and direction.

14. Cross product should be zero.

15. Dot product should be zero.


16. Scalar : Volume, mass speed, density, number of moles, angular frequency
Vector : Acceleration, velocity, displacement, angular velocity
17. Work and Current are scalar quantities.
Work done is given by the dot product of force and displacement. Since the dot product of two quantities
is always a scalar, work is a scalar physical quantity.
Current is described only by its magnitude. Its direction in space is not taken into account. Hence, it is a
scalar quantity.
18. Impulse
Impulse is given by the product of force and time. Since force is a vector quantity, its product with time
(a scalar quantity) gives a vector quantity.
19. (a) Meaningful
(b) Not Meaningful
(c) Meaningful
(d) Meaningful
(e) Meaningful
(f) Meaningful
Explanation :
(a) The addition of two scalar quantities is meaningful only if they both represent the same physical
quantity.
(b) The addition of a vector quantity with a scalar quantity is not meaningful.
(c) A scalar can be multiplied with a vector. For example, force is multiplied with time to give impulse.
(d) A scalar, irrespective of the physical quantity it represents, can be multiplied with another scalar
having the same or different dimensions.
(e) The addition of two vector quantities is meaningful only if they both represent the same physical
quantity.
(f) A component of a vector can be added to the same vector as they both have same dimension
20. (a) True
(b) False
(c) False
(d) True
(e) True
Explanation :
(a) The magnitude of a vector is a number. it is a scalar.
(b) Each component of a vector is also vector.
(c) Total path length is a scalar quantity, whereas displacement is a vector quantity. Hence, the total path
length is always greater than the magnitude of displacement. It becomes equal to the magnitude of
displacement only when a particle is moving in a straight line.
(d) It is because of the fact that the total path length is always greater than or equal to the magnitude of
displacement of a particle.
(e) Three vectors, which do not lie in a plane, cannot be represented by the sides of a triangle taken in
the same order.

E 43
JEE ( Main) - Physics ALLEN
CHECK YOUR GRASP EXERCISE-I
BASIC MATHS 8. The slope of graph in figure at point A, B and C
dy is mA, mB and mC respectively, then :
1. If y = logex + sinx + ex then is
dx
y
1 1
(1) + sinx + ex (2) – cosx + ex A
x x
B
1 1 C
(3) + cosx + ex (4) – sinx x
x x

d 100
2. (e ) = .......... (1) mA > mB > mC
dx
(2) mA < mB < mC
(1) e100 (2) 0
(3) 100 e99 (4) None of these (3) mA = mB = mC
(4) mA = mC < mB
d 9. The equation of graph shown in figure is
3. (sin 120°) = ............
dx y = 2x2. The slope of graph at point P is :
(1) cos 120° (2) 120 cos 120°
y
(3) 0 (4) None of these
4

The value of integral ⌠


dx
4.  is :- P(1, 2)
⌡ x
2
x
(1) 3 loge 2 (2) loge 2
(3) loge 4 (4) 2 loge8 (1) 1 (2) 2 (3) 3 (4) 4
5. Area bounded by curve y = sin x, with x-axis,
dy
π 10. At x = 0, value of is :
when x varies from 0 to is :- dx
2
(1) 1 unit (2) 2 units
y
(3) 3 units (4) 0
6. The radius of spherical bubble is changing with time.
Then rate of change of its volume is given by :
O x
dr 4
(1) 4πr2 (2) πr2
dt 3
8 8 dr
(3) πr2 (4) πr
3 3 dt
7. Which of the following statement is not correct (1) 0 (2) 1 (3) –1 (4) Infinite
for following straight line graph :- UNITS & DIMENSIONS
y
11. The ratio of one micron to one nanometre is
Line(1)
(1) 103 (2) 10–3 (3) 10–6 (4) 10–1
Line(2)
x 12. Temperature can be expressed as a derived quantity
in terms of which of the following.
(1) length and mass

(1) Line (2) has negative y intercept (2) mass and time
(2) Line (1) has positive y intercept (3) length, mass and time
(3) Line (2) has positive slope
(4) in terms of none of these
(4) Line (1) has negative slope
44 E
ALLEN Basic Mathematics
13. Which of the following is smallest unit of distance 23. A dimensionless quantity
(1) milimetre (2) angstrom (3) fermi (4) metre (1) never has a unit (2) always has a unit
14. Which of the following is not the unit of length (3) may have a unit (4) does not exist

(1) micron (2) light year (3) angstrom (4) radian 24. If the dimensions of a physical quantity are given
by MaLbTc, then the physical quantity will be :
15. One kilowatt hour is equal to -
(1) Force if a = 0, b = –1, c = – 2
(1) 3.6 × 106 joule (2) 3.6 × 104 joule
(3) 3.6 × 103 joule (4) 6 × 10–4 joule (2) Pressure if a = 1, b = – 1, c = – 2
16. Which of the following functions of A and B may (3) Velocity if a = 1, b = 0, c = – 1
be performed if A and B possess different
(4) Acceleration if a = 1, b = 1, c = – 2
dimensions
SIGNIFICANT FIGURES & ERROR IN MEASUREMENT
A
(1) (2) A + B (3) A – B (4) none
B 25. Significant figures in 3400 are-
17. The dimensional formula of angular velocity is - (1) 2 (2) 5 (3) 6 (4) 7
(1) MOLOT–1 (2) MLT–1
26. The percentage errors in the measurement of mass
(3) MOLOT1 (4) MLOT–2 and speed are 2% and 3% respectively. How much
18. Dimension of relative density is - will be the maximum error in the estimate of kinetic
(1) kg m–3 (2) ML–3 energy obtained by measuring mass and speed ?

(3) dimensionless (4) M2 L–6 (1) 11% (2) 8% (3) 5% (4) 1%


19. The dimensions of universal gravitational constant 27. The density of a cube is measured by measuring its
G are -
mass and the length of its side. If the maximum
–2 3 –2
(1) [M L T ] (2) [M L T ] errors in the measurement of mass and length are
(3) [M–1 L3 T–2] (4) [M–1 L T–2] 4% and 3% respectively, the maximum error in

20. The equation of state of some gases can be the measurement of the density is -

 a  (1) 9% (2) 13% (3) 12% (4) 7%


expressed as  P + 2  (V − b) = RT , where P is the
V
pressure, V is the volume, T is the absolute 28. The error in the measurement of radius of a sphere
temperature and a, b and R are constants. The is 0.1% . The error in the measurement of volume is
dimensions of 'a' are
(1) 0.1% (2) 0.3% (3) 0.5% (4) 0.8%
(1) [ML5 T–2] (2) [M L–1 T–2]
29. An experiment measures quantities a, b and c, and
(3) [L3] (4) [L6] X is calculated from X = ab2/c3. If the percentage
21. From the following pairs, choose the pair that does error in a, b and c are ±1%, ±3% and ±2%
not have identical dimensions respectively, the percentage error in X will be –

(1) impulse and momentum (1) ±13% (2) ±7% (3) ±4% (4) ±1%
30. If error in measuring diameter of a circle is 4%, the
(2) work and torque error in the radius of the circle would be
(3) moment of inertia and moment of force (1) 2% (2) 8% (3) 4% (4) 1%
31. If a, b, c are the percenatage errors in the
(4) angular momentum and Planck's constant
measurment of A, B and C, then percentage error
22. A force F is given by F = at + bt2, where t is time. in ABC would be approximately -
The dimension of a and b are - (1) abc (2) a + b + c
(1) [M L T–3] and [M L T–4] (2) [M L T–4] and [M L T–3] a b c
(3) ab + bc + ac (4) + +
b c a
(3) [M L T–1] and [M L T–2] (4) [M L T–2] and [M L T0]

E 45
JEE ( Main) - Physics ALLEN
→ →
32. The radius of a disc is 1.2 cm. Its area according The vectors and b are such that
40. a
to idea of significant figures, will be given by– → → → → →
2 2 |a + b|=|a − b|, then the angle between a and
(1) 4.5216 cm (2) 4.521 cm
2 2 →
(3) 4.52 cm (4) 4.5 cm b will be :–
33. The area of a rectangle of size 1.23 × 2.345 cm is
π π
(1) (2) π (3) (4) zero
(1) 2.88 cm 2
(2) 2.884 cm 2
3 2
(3) 2.9 cm2 (4) 2.88435 cm2 41. Force 3N, 4N and 12N act at a point in mutually
perpendicular directions. The magnitude of the
34. The significant digits in 200.40 are resultant force is :–
(1) 4 (2) 5 (3) 2 (4) 3 (1) 19 N (2) 13 N (3) 11 N (4) 5 N
→ → → →
35. The radius of a thin wire is 0.16 mm. The area of 42. If |A + B|=|A|=|B| then angle between A and B
cross-section taking significant figure into
will be :–
consideration in square millimetres is
(1) 90° (2) 120° (3) 0° (4) 60°
(1) 0.0804 (2) 0.080
43. ˆ are :-
The direction cosines of a vector ˆi + ˆj + 2 k
(3) 0.08 (4) 0.080384
1,1, 1 , 1 ,1
36. A cube has a side of length 1.2 ×10–2 m. Calculate (1) 1 (2)
its volume – 2 2 2 2 2

(1) 1.7 × 10–6 m3 (2) 1.73 × 10–6 m3 1,1, 1 1 , 1 , 1


(3) (4)
2 2 2 2 2 2
(3) 1.70 × 10–6 m3 (4) 1.728 × 10–6 m3
44. A physical quantity which has a direction :–
37. The length, breadth and thickness of a strip are (1) must be a vector (2) may be a vector
(10.0±0.1) cm (3) must be a scalar (4) none of the above
45. ˆ and
The angle between two vectors (2iˆ + 3jˆ + k)
(1.00±0.01) cm and
(0.100± 0.001) cm ˆ is :–
( −3iˆ + 6k)
respectively. The most probable error in its volume (1) 0° (2) 45°
will be (3) 60° (4) 90°
(1) ± 0.03 cm3 (2) ± 0.111 cm3 →
46. Let A = ˆiA cos θ + ˆjA sin θ , be any vector. Another
(3) ± 0.012 cm3 (4) None of these
→ →
vector B which is normal to A is :-
VECTOR
(1) ˆiB cos θ + ˆjB sin θ (2) ˆiB sin θ + ˆjB cos θ
38. If a unit vector is represented by 0.5iˆ − 0.8ˆj + ckˆ ,
then the value of 'c' is :– (3) ˆiB sin θ − ˆjB cos θ (4) ˆiA cos θ − ˆjA sin θ
(1) 1 (2) 0.11 →
47. The torque of a force F = 2iˆ + ˆj + 4kˆ acting at
(3) 0.01 (4) 0.39 →
the point r = 7iˆ + 3ˆj + kˆ is –
→ → → → → →
39. If A + B = C and |A| = |B|=|C| then the angle (1) 14iˆ − 38ˆj + 16kˆ
→ →
between A and B is :– (2) 4iˆ + 4ˆj + 6kˆ

(1) 45° (2) 60° (3) −14iˆ + 38ˆj − 16kˆ

(3) 90° (4) 120° (4) 11iˆ − 26ˆj + kˆ

46 E
ALLEN Basic Mathematics

→ → → 55. The minimum number of vectors of equal


48. If vectors P , Q and R have magnitudes 5, 12 mangitude required to produce a zero resultant is -
→ → →
(1) 2 (2) 3
and 13 units and P + Q = R , the angle between
(3) 4 (4) more than 4
→ →
Q and R is :– 56. How many minimum number of coplanar vectors
−1 5 −1 5
having different magnitudes can be added to give
(1) cos (2) cos zero resultant :–
12 13

−1 12 −1 2 (1) 2 (2) 3
(3) cos (4) cos
13 13
(3) 4 (4) 5
→ → → →
49. If P.Q = PQ , then angle between P and Q is :–
57. Vector sum of two forces of 10N and 6N cannot
(1) 0° (2) 30° be :-
(3) 45° (4) 60°
→ → (1) 4N (2) 8N
50. A and B are two vectors. Now indicate the wrong (3) 12N (4) 2N
statement in the following :–
→ → → → → → → →
(1) A. B = B. A 58. The value of (A + B).(A × B) is :–
→ → → →
(1) 0 (2) A2 – B2
(2) A + B = B+ A
→ → → → (3) A2 + B2 + 2AB (4) none of these
(3) A× B = B× A
59. What happens, when we multiply a vector by (– 2) ?
→ → → →
(4) A× B = − B× A (1) direction reverses and unit changes
→ →
51. If three vectors satisfy the relation A .B = 0 and (2) direction reverses and magnitude is doubled
→ → → (3) direction remains unchanged and unit changes
A.C = 0 , then A can be parallel to :–
(4) none of these
→ →
(1) C (2) B
60. Which of the following vector identities is false ?
→ → → → r r r r r r r r
(3) B × C (4) B.C (1) P + Q = Q + P (2) P + Q = Q × P
r r r r r r r r
52. If a vector 2$i + 3 $j + 8 k$ is perpendicular to the (3) P.Q = Q.P (4) P × Q ≠ Q × P
r r
vector 4 $j − 4 $i + αk$ , then the value of α is :– 61. If P = KQ (Here K is constant) then :-
1 r r
(1) –1 (2) (1) P | | Q
2
1 r r
(3) − (4) 1 (2) P Q
2
r r
→ → (3) P ⊥ Q
53. If the angle between the vectors A and B is θ,
→ → →
(4) Both (1) and (2)
the value of the product (B× A).A is equal to :–
62. Which of the followig group of concurrent forces
(1) BA2 cosθ (2) BA2 sinθ
may be in equilibrium (R = 0)?
(3) BA2 sinθ cosθ (4) zero
A B C
54. Correct relation is :–
(1) 10 20 40
(1) $j × k$ = $i (2) $i . $i = 0 (2) 3 5 1

(3) $j × $j = 1 (4) k$ . $i = 1 (3) 20 20 20


(4) 40 30 5

E 47
JEE ( Main) - Physics ALLEN
63. With respect to a rectangular cartesian coordinate 65. The position vectors of points A, B, C and D are
system, three vectors are expressed as r r
r r r A = 3iˆ + 4 ˆj + 5kˆ , B = 4iˆ + 5ˆj + 6kˆ ,
a = 4iˆ − ˆj, b = −3iˆ and c = − kˆ where ˆi, ˆj, kˆ are
r r
unit vector then r̂ along the direction of sum of C = 7iˆ + 9ˆj + 3kˆ and
uuur D = 4iˆ + 6 ˆj , then
these vector is :-
uuur
displacement vectors AB and CD are :-

(1) r̂ = (i − j − k)
1 ˆ ˆ ˆ
(2) r̂ = (i + j − k)
1 ˆ ˆ ˆ
(1) Same (2) Parallel
3 2
(3) Perpendicular (4) Antiparallel
(3) r̂ = ( i − j + k ) (4) r̂ = 1 ( ˆi + ˆj + kˆ )
1 ˆ ˆ ˆ
3 2
r
64. Write the vector F in terms of its component :-
y →
F 20 2

45°
x

r r
(1) F = 10iˆ + 10 ˆj (2) F = 20iˆ + 20 ˆj
r r
(3) F = 30iˆ + 30 ˆj (4) F = 40iˆ + 40 ˆj

CHECK YOUR GRASP ANSWER KEY Exercise-I


Que. 1 2 3 4 5 6 7 8 9 10
Ans. 3 2 3 2 1 1 4 2 4 4
Que. 11 12 13 14 15 16 17 18 19 20
Ans. 1 4 3 4 1 1 1 3 3 1
Que. 21 22 23 24 25 26 27 28 29 30
Ans. 3 1 3 2 1 2 2 2 1 3
Que. 31 32 33 34 35 36 37 38 39 40
Ans. 2 4 1 2 2 1 1 2 4 3
Que. 41 42 43 44 45 46 47 48 49 50
Ans. 2 2 3 2 4 3 4 3 1 3
Que. 51 52 53 54 55 56 57 58 59 60
Ans. 3 3 4 1 1 2 4 1 2 2
Que. 61 62 63 64 65
Ans. 4 3 1 2 4

48 E
ALLEN Basic Mathematics

BRAIN TEASERS EXERCISE-II


BASIC MATHS 7. Magnitude of slope i.e. steepness of graph shown
in figure
x2 dy
1. If y = then is :- y
(x + 1) dx

x(3x + 2) − x(x + 2)
(1) (2)
(x + 1)2 (x + 1)2
x(x + 2) x(x + 2) O
x
(3) (4)
(x + 1)2 (x + 1)
(1) First increases and then decreases
dy
2. 3
If y = x cosx then = ........ (2) First decreases and then increases
dx
(3) Decreases continuously
(1) x2(3cosx – x sinx) (2) x2(3cosx + xsinx) (4) Increases continuously
(3) 3x ⋅cosx + x sinx
2 3
(4) None of these
UNITS AND DIMENSIONS
2
d y 8. The fundamental unit which has same power in
3. If y = sinx + cosx then is :-
dx 2 the dimensional formula of surface tension and
(1) sinx – cosx (2) cosx – sinx viscosity is
(3) –(sinx + cosx) (4) None of these (1) mass (2) length
4. If radius of a spherical bubble starts to increase
(3) time (4) none
with time t as r = 0.5t. What is the rate of change
of volume of the bubble with time t = 4s ? 9. The equation of a wave is given by Y = A sin
(1) 8π units/s (2) 4π units/s x 
ω  − k  where ω is the angular velocity and v
(3) 2π units/s (4) π units/s v 
5. If velocity v varies with time t as v = 2t2, then the is the linear velocity. The dimension of k is
plot between v and t2 will be given as : (1) LT (2) T
v v (3) T–1 (4) T2
10. The time dependence of a physical quantity P is
(1) (2) given by P = P0 exp (–α t2), where α is a constant
t
2 2
t and t is time. The constant α
(1) dimensionless
v v
(2) has dimensions T–2
(3) has dimensions of P
(3) (4)
t
2 2
t (4) has dimensions T2
11. Density of wood is 0.5 g/cc in the CGS system of
dy units. The corresponding value in MKS units is
6. At point P, the value of is : (1) 500 (2) 5
dx
(3) 0.5 (4) 5000
y
12. In a particular system the units of length, mass and
time are chosen to be 10 cm, 10 g and 0.1 s
P respectively. The unit of force in this system will
be equal to
x
(1) 0.1 N (2) 1 N

(1) Zero (2) Positive (3) 10 N (4) 100 N


(3) Negative (4) Infinite

E 49
JEE ( Main) - Physics ALLEN
13. Match the following : 20. If pressure P, velocity V and time T are taken as
(A) Angular momentum (1) M–1 L2 T–2 fundamental physical quantities, the dimensional
(B) Torque (2) M T–2 formula of the force is-
2 2 –1 2 –2
(1) PV T (2) P V T
(C) Gravitational constant (3) M L2 T–2
2 –1 2
2 –1
(3) PVT (4) P VT
(D) Tension (4) M L T
(1) (C) – 2, (D) –1 (2) (A) – 4, (B) –3 21. If the velocity of light c, gravitational constant G
and planck’s constant h be taken as fundametal
(3) (A) –3, (C) –1 (4) (B) – 2, (A) –1 units the dimension of mass in the new system
14. The velocity v of a particle at time t is given by will be -
1/2 1/2 1/2
b (1) c h G
v = at + , where a, b and c are constants.
t+c (2) c
1/2 1/2
h G
–1/2
The dimensions of a, b and c are respectively :- –3/2 1/2 1/2
(3) c h G
(1)LT–2, L and T (2) L2 ,T and LT2 –5/2 1/2 1/2
(3) LT2, LT and L (4) L, LT and T2 (4) c h G
22. In a relation F = a sink1x + b sink2t, F, x and t
15. Which of the following quantity is unitless
denote the force, distance and time respectively.
(1) velocity gradient (2) pressure gradient Units of k1 and k2 are respectively as –
(3) displacement gradient (4) force gradient –1 –1
(1) metre, s (2) metre , s
16. Which of the following is incorrect statement –1 –1
(3) metre , s (4) metre, s
(1) a dimensionally correct equation may be correct 23. The number of particles crossing the unit area
(2) a dimensionally correct equation may be perpendicular to the x-axis per unit time is given
incorrect
 n2 − n1 
(3) a dimensionally incorrect equation may be by N = −D  x − x  when n 1 and n 2 are the
 2 1 
correct
number of particles per unit volume for the values
(4) a dimensionally incorrect equation is incorrect
of x meant to be x1 and x2 respectively. What is the
17. A unitless quantity dimensional formula for the diffusion constant D ?
(1) does not exist (1) M0LT2 (2) M0L2 T4
(2) always has a nonzero dimension (3) M0LT-3 (4) M0L2T-1
(3) never has a nonzero dimension SIGNIFICANT FIGURES & ERROR IN MEASUREMENT
(4) may have a nonzero dimension 24. The pressure on a square plate is measured by
18. When a wave transverses in a medium, the measuring the force on the plate and the length of
displacement of a particle located at distance x at the sides of the plate. If the maximum error in the
time t is given by y = a sin (bt – cx) where a, b and measurement of force and length are respectively
c are constants of the wave. The dimension of b/c 4% and 2%, the maximum error in the
are same as that of : measurement of pressure is –
(1) wave velocity (1) 1% (2) 2%
(3) 6% (4) 8%
(2) wavelength
25. When a copper sphere is heated, maximum
(3) wave amplitude percentage change will be observed in–
(4) wave frequency (1) radius (2) area
19. Which of the following does not have the (3) volume (4) none of these
dimensions of force ? 26. A scientist performs an experiment in order to
(1) potential gradient measure a certain physical quantity and takes 100
observations. He repeats the same experiment and
(2) energy gradient
takes 400 observations, by doing so –
(3) weight (1) The possible error remains same
(4) rate of change of momentum (2) The possible error is doubled
(3) The possible error is halved
(4) The possible error is reduced to one fourth

50 E
ALLEN Basic Mathematics
27. A quantity is represented by X = Ma Lb Tc. The 36. A student per for ms an experiment for
percentage error in measurement of M, L and T
are α%, β % and γ % respectively. The percentage  4π2 l 
determination of g  = 2  . The error in length
error in X would be  T 
(1) (α a + βb + γc) % (2) (αa – βb + γc) % l is ∆l and in time T is ∆T and n is number of
(3) (α a – βb– γc) % (4) None of these times the reading is taken. The measurement of g
28. If error in measuring diameter of a circle is 4 %, is most accurate for ?
the error in circumference of the circle would be :- ∆l ∆T n
(1) 2% (2) 8% (3) 4% (4) 1%
3
(1) 5mm 0.2 s 10
29. The volume of a sphere is 1.76 cm . The volume
(2) 5mm 02 s 20
of 25 such spheres taking into account the
significant figure is- (3) 5mm 0.1 s 10
(1) 0.44 × 102 cm3 (2) 44.0 cm3 (4) 1mm 0.1 s 50
(3) 44 cm3 (4) 44.00 cm3 VECTOR
30. What is the fractional error in g calculated from
37. A force vector applied on a mass is represented as
l →
T = 2π ? Given that fractional errors in T and
g F = 6iˆ − 8ˆj + 10kˆ newton and the mass accelerates
l are ± x and ± y respectively. with 1m/s2. What will be the mass of the body ?
(1) x + y (2) x – y (3) 2x + y (4) 2x – y)
(1) 10 2 kg (2) 2 10 kg
V
31. The resistance is R = where V = 100 ± 5 Volts (3) 10 kg (4) 20 kg
I
and I = 10 ± 0.2 amperes. What is the total error 38. If the sum of two unit vectors is a unit vector, then
in R ? the magnitude of their difference is :–
(1) 5% (2) 7%
1
 5 (1) 2 (2) 3 (3)
2
(4) 5
(3) 5.2% (4)   %
2
39. The sum of magnitudes of two forces acting at a
32. The length, breadth and thickness of a strip are
point is 16N. If the resultant force is 8N and its
(10.0 ± 0.1)cm, (1.00 ± 0.01)cm and (0.100 ±
direction is perpendicular to smaller force, then the
0.001) cm respectively. The most probable error
forces are :–
in its volume will be
(1) 6N and 10N (2) 8N and 8N
(1) ±0.03 cm3 (2) ±0.111 cm3
(3) 4N and 12N (4) 2N and 14N
(3) ±0.012 cm3 (4) none of these

33. The external and internal radius of a hollow cylinder 40. For a body, angular velocity ω = $i –2 $j + 3 k$ and
are measured to be (4.23 ± 0.01) cm and

(3.89 ± 0.01) cm. The thickness of the wall of radius vector r = $i + $j + k$ , then its velocity is:–
the cylinder is :-
(1) (0.34 ± 0.02)cm (2) (0.17 ± 0.02)cm (1) –5 $i + 2 $j + 3 k$ (2) –5 $i + 2 $j – 3 k$
(3) (0.17 ± 0.01)cm (4) (0.34 ± 0.01)cm
(3) –5 $i – 2 $j + 3 k$ (4) –5 $i – 2 $j – 3 k$
34. A wire has a mass (0.3 ± 0.003) g, radius
(0.5 ± 0.005) mm and length (6 ± 0.06) cm. The 41. The unit vector parallel to the resultant of the
maximum percentage error in the measurement → →
of its density is– vectors A = 4iˆ + 3ˆj + 6kˆ and B = −ˆi + 3ˆj − 8kˆ
(1) 1 (2) 2 (3) 3 (4) 4
is :-
35. A wire has a mass (0.3 ± 0.003)g, radius
(0.5 ± 0.005) mm and length (6 ± 0.06) cm. The 1 ˆ ˆ 1 ˆ ˆ
(1) (3i + 6ˆj − 2k) (2) (3i + 6ˆj + 2k)
maximum percentage error in the measurement 7 7
of its density is –
1 ˆ ˆ 1 ˆ ˆ
(1) 1 (2) 2 (3) 3 (4) 4 (3) (3i + 6ˆj + 2k) (4) (3i + 6ˆj − 2k)
49 49
E 51
JEE ( Main) - Physics ALLEN
→ → 48. Following sets of three forces act on a body. Whose
42. A vector A points vertically upward and B points resultant cannot be zero ?
→ →
towards north. The vector product A× B is :– (1) 10, 10, 10 (2) 10, 10, 20

(1) null vector (2) along west (3) 10, 20, 20 (4) 10, 20, 40

(3) along east (4) vertically downward → → → →


49. Given that P + Q = P − Q . This can be true when :-

43. ˆ
If force is F = (60iˆ + 15ˆj − 3k)N and velocity → →
(1) P = Q

ˆ m/s, then instantaneous power
v = (2iˆ − 4ˆj + 5k) →
(2) Q is a null vector
is :-
→ →
(1) 195 watt (2) 45 watt. (3) neither P nor Q is a null vector
(3) 75 watt (4) 100 watt → →
(4) P is perpendicular to Q
44. The vector sum of two forces is perpendicular to
→ →
→ → →
their vector difference. In that case, the force : 50. Two vectors A and B are such that A + B = C
(1) Are equal to each other. and A 2 + B 2 = C 2 . Which of the following
statements, is correct :–
(2) Are equal to each other in magnitude. → →
(1) A is parallel to B
→ →
(3) Are not equal to each other in magnitude.
(2) A is anti-parallel to B
→ →
(4) Cannot be predicted. (3) A is perpendicular to B
→ →
→ → → → → → (4) A and B are equal in magnitude
45. If |A × B|= 3 A .B , then the value of |A + B| is :-
51. Area of a parallelogram, whose diagonals are
3 $i + $j − 2k$ and $i − 3 $j + 4k$ will be :–
FG
(1) A + B +
2 2 AB IJ 1/ 2

H 3K
(2) A + B (1) 14 unit (2) 5 3 unit

(3) 10 3 unit (4) 20 3 unit


→ →
(3) (A2+B2 + 3 AB)1/2 (4) (A2+B2+AB)1/2 52. If A = 3iˆ + 4ˆj and B = 6iˆ + 8jˆ and A and B are
→ →
46. Square of the resultant of two forces of equal the magnitudes of A and B respectively, then
magnitude is equal to three times their product. which of the following is not true ?
The angle between them is :-
→ → A 1
(1) A × B = 0 (2) =
(1) 0° (2) 45° (3) 60° (4) 90° B 2
→ →
47. The x-component of the resultant of several vectors- (3) A.B = 40 (4) none of these
(a) is equal to the sum of the x-components of the → →
53. Two vectors P and Q are inclined to each other
vectors
at angle θ. Which of the following is the unit vector
(b) May be smaller than the sum of the magnitudes
→ →
of the vectors perpendicular to P and Q :–
(c) May be greater than the sum of the magnitudes → →
P× Q ˆ
(2) P̂ × Q
of the vectors
(1)
(d) May be equal to the sum of the magnitudes of P.Q sinθ
the vectors
∧ →
(1) a, c, d (2) a, b, c (3) a, b, d (4) b, c, d ˆ
P̂ × Q P× Q
(3) PQ sin θ (4)
PQ sin θ
52 E
ALLEN Basic Mathematics
→ → → → → → → →
54. The projection of A on B is :– 60. If A × B = 0 and B× C = 0 , then the angle

∧ ∧ → →
→ → → ∧ → →
between A and C may be :–
(1) A. B (2) A. B (3) B. A (4) A. B
55. At what angle must the two forces (x + y) and (x – y) π
(1) 0 (2)
4
act so that the resultant may be (x 2 + y 2 ) :–
π
− (x2 + y2 ) − 2(x2 − y2 ) (3) (4) none of these
(1) cos–1 (2) cos–1 2
2(x2 − y2 ) x2 + y2
→ → →
− (x2 + y2 ) (x − y )
2 2 61. The resultant of A and B is perpendicular to A .
(3) cos–1 (4) cos–1
x2 − y2 x2 + y2 → →
What is the angle between A and B ?
→ → → →
56. Given that A + B + C = 0 . Out of these three
-1  A  -1  A
vectors two are equal in magnitude and the (1) cos   (2) cos  − 
B B
magnitude of the third vector is 2 times as that
of either of the two having equal magnitude. Then -1  A  -1  A
(3) sin   (4) sin  − 
the angles between vectors are given by :– B B
(1) 30°, 60°, 90° (2) 45°, 45°, 90°
→ → →
(3) 45°, 60°, 90° (4) 90°, 135°, 135° 62. The resultant of A and B is R1 . On reversing the

→ → → → →
57. The resultant of two vectors P and Q is R . If vector B , the resultant becomes R 2 . The value

Q is doubled then the new resultant vector is of R12 + R 22 is :–
→ →
perpendicular to P . Then magnitude of R is :–
(1) A2 + B2 (2) A2 – B2

P 2 − Q2 (3) 2(A2 + B2) (4) 2(A2 – B2)


(1) (2) Q
2PQ 63. A displacement vector, at an angle of 30° with
y-axis has an x-component of 10 units. Then the
P P+Q magnitude of the vector is-
(3) (4) P − Q
Q (1) 5.0 (2) 10
(3) 11.5 (4) 20
58. The component of 3$i + 4$j along $i + $j :–

64. What is the angle between vector A = $i + $j + 2 k$
1 ˆ ˆ 3ˆ ˆ
(1) ( j + i) (2) ( j + i) and the Z-axis-
2 2
(1) 0° (2) 45°
5 7
(3) (ˆj + ˆi) (4) (ˆj + ˆi) (3) 60° (4) 90°
2 2
65. Find out the unit vector perpendicular to both
→ → →
59. If A + B = C and A + B = C, then the angle vectors $i − $j + k$ and $i + $j + k$
→ →
between A and B is :– − $i + k$
(1) $i + $j (2)
π 2
(1) 0 (2)
4
π (3) $j + k$ (4) ($j + $i ) 2
(3) (4) π
2

E 53
JEE ( Main) - Physics ALLEN
66. A parallelopiped has edges described by the vector r r r
69. Figure shows three vectors a, b and c . If
$i + 2$j , 4 $j , $j + 3k$ . Then the volume is :- uuur uuur
RQ = 2PR , which of the following relation is
(1) 11 (2) 12 (3) 15 (4) 28 correct :-
→ →
r r r
67. Find the component of r in the direction of a :- (1) 2a + c = 3b Q

→ → → → → → r r r →
a
( r .a) a ( r .a) a (2) a + 3c = 2b → R
(1) (2) b
a² a r r r →
(3) 3a + c = 2b O
c
P
→ → →
( r × a) a r r r
(3) (4) None of above (4) a + 2c = 3b

r r r
68. From the fig., the correct relation is - a+b r
70. If r r = 1, then angle between a and b is :-
→ → → →
→ a−b
(1) A + B+ E = 0 C (1) 0° (2) 45° (3) 90° (4) 60°
O N
→ → → → → If â + 2bˆ and 5aˆ − 4bˆ are perpendicular to each
(2) C − D = − A D → B 71.
A
other, find angle between â and b̂ :-
→ → → → M → P (1) 60° (2) 90° (3) 45° (4) None
(3) B+ E − C = − D E
(4) All of the above 72. If vector ˆi + ˆj − kˆ and 2iˆ + 2 ˆj + λkˆ are parallel
than find out the value of λ :-
(1) –1 (2) 1
(3) –2 (4) 2

BRAIN TEASERS ANSWER KEY Exercise-II


Que. 1 2 3 4 5 6 7 8 9 10
Ans. 3 1 3 1 1 1 2 1 2 2
Que. 11 12 13 14 15 16 17 18 19 20
Ans. 1 1 2 1 3 3 3 1 1 1
Que. 21 22 23 24 25 26 27 28 29 30
Ans. 2 2 4 4 3 4 1 3 2 3
Que. 31 32 33 34 35 36 37 38 39 40
Ans. 2 1 1 4 4 4 1 2 1 1
Que. 41 42 43 44 45 46 47 48 49 50
Ans. 1 2 2 2 4 3 3 4 2 3
Que. 51 52 53 54 55 56 57 58 59 60
Ans. 2 3 2 2 1 4 2 4 1 1
Que. 61 62 63 64 65 66 67 68 69 70
Ans. 2 3 4 2 2 2 1 4 4 3
Que. 71 72
Ans. 1 3

54 E
ALLEN Basic Mathematics

PREVIOUS YEARS QUESTIONS EXERCISE-III


1. Identify the pair whose dimensions are equal- 9. The electrical resistance 'R' of a conductor of length
[AIEEE - 2002] ρl
'l' and area of cross secion 'a' is given by R =
(1) Torque and work a
(2) stress and energy where 'ρ' is the electrical resistivity. What is the
dimensional formula for electrical conductivity 'σ'
(3) Force and stress
which is the reciprocal of resistivity ?
(4) Force and work
[AIEEE - 2012 (Online)]
2. Which one of the following represents the correct
dimensions of the coefficient of viscosity ? (1) M–1 L–3 T3 A2 (2) ML–3T–3A2
(3) M–2L3T2A–1 (4) ML3T–3A–2
[AIEEE - 2004]
(1)[ML–1T–2] (2) [MLT–1] 10. The current voltage relation of diode is given by
(3) [ML–1T–1] (4) [ML–2T–2] I = (e1000V/T – 1) mA, where the applied voltage
→ → → → V is in volts and the temperature T is in Kelvin.
3. If A × B = B × A , then the angle between A and If a student makes an error measuring
B is- [AIEEE - 2004] ± 0.01V while measuring the current of 5 mA at
(1) π (2) π/3 300 K, what will be error in the value of current
in mA ? [JEE (MAIN) - 2014]
(3) π/2 (4) π/4
(1) 0.5 mA (2) 0.05 mA

4. A force F = ( 5iˆ + 3ˆj + 2kˆ ) N is applied over a (3) 0.2 mA (4) 0.02 mA
particle which displaces it from its origin to the 11. In the following 'I' refers to current and other

point r = ( 2iˆ − ˆj ) m. The work done on the symbols have their usual meaning. Choose the
option that corresponds to the dimensions of
particle in joules is- [AIEEE - 2004]
electrical conductivity : [JEE (MAIN) - 2016 (Online)]
(1) –7 (2) +7
(3) +10 (4) +13 (1) M –1 L –3 T 3I (2) M –1 L 3T 3 I

5. Out of the following pairs, which one does not have (3) ML–3 T –3 I 2 (4) M –1 L –3 T 3I 2
identical dimensions ? [AIEEE - 2005]
12. The following observations were taken for
(1) Angular momentum and Planck's constant determining surface tensiton T of water by capillary
(2) Impulse and momentum method : [JEE (MAIN) - 2017]
(3) Moment of inertia and moment of a force
Diameter of capilary, D = 1.25 × 10–2 m
(4) Work and torque
6. The respective number of significant figures for the rise of water, h = 1.45 × 10–2 m
numbers 23.023, 0.0003 and 2.1 × 10–3 are:- Using g = 9.80 m/s2 and the simplified relation
[AIEEE - 2010]
rhg
(1) 4, 4, 2 (2) 5, 1, 2 T= × 103 N/m, the possible error in surface
2
(3) 5, 1, 5 (4) 5, 5, 2
tension is closest to :
7. If 400 Ω of resistance is made by adding four100
Ω resistance of tolerance 5%, then the tolerance (1) 2.4% (2) 10%
of the combination is : [AIEEE - 2011]
(3) 0.15% (4) 1.5%
(1) 20% (2) 5%
13. Expression for time in terms of G (universal
(3) 10% (4) 15% gravitational constant), h (Planck constant) and c
8. Resistance of a given wire is obtained by measuring (speed of light) is proportional to :
the current flowing in it and the voltage difference [JEE (MAIN) - 2019, January]
applied across it. If the percentage errors in the
Gh hc 5
measurement of the current and the voltage (1) (2)
difference are 3% each, then error in the value of c3 G
resistance of the wire is :- [AIEEE - 2012]
c3 Gh
(1) 3% (2) 6% (3) zero (4) 1% (3) (4)
Gh c5
E 55
JEE ( Main) - Physics ALLEN
14. The density of a material in SI units is 17. The diameter and height of a cylinder are measured
128 kg m–3. In certain units in which the unit of by a meter scale to be 12.6 ± 0.1 cm and 34.2
length is 25 cm and the unit of mass is ± 0.1 cm, respectively. What will be the value of
50 g, the numerical value of density of the material its volume in appropriate significant figures ?
is : [JEE (MAIN) - 2019, January] [JEE (MAIN) - 2019, January]
(1) 410 (2) 640 (3) 16 (4) 40 (1) 4260 ± 80 cm3
15. In the cube of side 'a' shown in the figure, the vector
(2) 4300 ± 80 cm3
from the central point of the face ABOD to the
central point of the face BEFO will be: (3) 4264.4 ± 81.0 cm3
[JEE (MAIN) - 2019, January] (4) 4264 ± 81 cm3
z r r
18. Two vectors A and B have equal magnitudes.
r r
B E The magnitude of ( A + B) is 'n' times the
r r r
A
H ( )
magnitude of A – B . The angle between A and
G a r
B is : [JEE (MAIN) - 2019, January]
O F y
 n2 − 1   n −1
a  2 
(1) sin–1 (2) cos–1  
D a  n + 1  n + 1
x
 n2 − 1   n −1
1
2
(
(1) a ˆi − kˆ ) 1
2
( )
(2) a ˆj − ˆi (3) cos–1  2 
 n + 1
(4) sin–1  
 n + 1

(3)
1 ˆ ˆ
2
(
a k−i ) (4)
1 ˆ ˆ
2
(
a j−k ) 19. If speed (V), acceleration (A) and force (F) are
considered as fundamental units, the dimension of
16. Two forces P and Q of magnitude 2F and 3F, Young's modulus will be :-
respectively, are at an angle θ with each other. If
[JEE (MAIN) - 2019, January]
the force Q is doubled, then their resultant also gets
doubled. Then, the angle is : (1) V–2 A2F2 (2) V–4A2F
[JEE (MAIN) - 2019, January]
(3) V–4A–2F (4) V–2A2F–2
(1) 30° (2) 60° (3) 90° (4) 120°

PREVIOUS YEARS QUESTIONS ANSWER KEY Exercise-III


Que. 1 2 3 4 5 6 7 8 9 10
Ans. 1 3 1 2 3 2 2 2 1 3
Que. 11 12 13 14 15 16 17 18 19
Ans. 4 4 4 4 2 4 1 3 2
56 E

You might also like